MCQ Flashcards

1
Q

What is true about the sesamoids?

A

the tibial sesamoid is larger

Blood supply - medial plantar artery and plantar arch

Sesamoid complex transmits 50% of the body weight

How well did you know this?
1
Not at all
2
3
4
5
Perfectly
2
Q

What are the surgical options for a burst fracture?

A

Anterior decompression with stabilization

Retropulsion with need for decompression

corepectomy with strut grafting

Posterior decompression with fusion

Instrumentation with distraction, get indirect decompression of the canal

Posterior fusion

Progressive kyphotic deformity

How well did you know this?
1
Not at all
2
3
4
5
Perfectly
3
Q

Compare two incision distal biceps

A

both can get PIN

same HO

better flexion with two incision

less LABCN with two incision

How well did you know this?
1
Not at all
2
3
4
5
Perfectly
4
Q

What are approapriate antibiotics for open wounds

A
  • Gustilo 1
    • cefazolin
    • clinda if allergy
  • Gustilo 2/3
    • cefazolin
    • gentamycin
    • Farm wounds - penicillin or flagyl
  • Freshwater (A hydrophilia)
    • cipro or ceftazidime
  • Salt water (vibrio)
    • Doxycycline
    • ceftaxidime or cipro
How well did you know this?
1
Not at all
2
3
4
5
Perfectly
5
Q

What is true about NF2?

A
  • less common
  • bilateral acoustic neuroma
  • no scoliosis
How well did you know this?
1
Not at all
2
3
4
5
Perfectly
6
Q

What are the indications for treatment of a pediatric proxiaml humerus fracture?

A
  • Neer-Horowitz classification
    • type I is minimally displaced (< 5mm)
    • type II is displaced less than 1/3 of the shaft width
    • type III is displaced greater than 1/3 and less than 2/3 the shaft width
    • type IV is displaced greater than 2/3 the shaft width
  • > 6 with the following displacement
    • >50% displaced
    • > 45 deg angulated
    • open
    • dysvascular
How well did you know this?
1
Not at all
2
3
4
5
Perfectly
7
Q

Sites of compression of the ulnar nerve at the cubital tunnel

A

Arcade of struthers
Medial intermuscular septum
Medial epicondyle
Osborne’s ligament
Between heads of FCU (fasica)
Aponeurosis of FDS

How well did you know this?
1
Not at all
2
3
4
5
Perfectly
8
Q

What structure causes retrograde ejaculation in the anterior approach to the lumbar spine?

A

Superior hypogastric plexus

How well did you know this?
1
Not at all
2
3
4
5
Perfectly
9
Q

What are the releases for a valgus knee?

A

osteophytes

Capsule

IT (extension)

popliteus (flexion)

LCL

How well did you know this?
1
Not at all
2
3
4
5
Perfectly
10
Q

What is your approach to factor replacement in a hemophiliac

A
  • 2 hours pre op infuse with clotting factor calculated to attain 100% activity of normal
  • Then maintained on continuous infusion of factor to maintain levels at >60% throughout the procedure
    • Thereafter, levels are checked every 1 or 2 days.
    • A 60% level is maintained by continuous infusion until the patient is discharged.
  • Home infusion of bolus doses is used to maintain levels of 30% to 60% for at least 2 weeks postoperatively.
  • If vigorous physical therapy is needed the patient is infused to a 30% level just before therapy
  • Complications
    • hemarthorosis at 4-7 days if it occurs
    • infection slightly higher
    • risk of stiffness so no concern for instability
How well did you know this?
1
Not at all
2
3
4
5
Perfectly
11
Q

Complications of talar neck fractures

A
  • Subtalar OA
  • AVN
  • malunion
  • In that order, no matter the Hawkins classification, but the risk of each one will go up with subsequent grade
  • Nonunion is a very low risk
How well did you know this?
1
Not at all
2
3
4
5
Perfectly
12
Q

What is the incidence of bilateral slip in a SCFE?

A

General population - 20%

Renal osteodystrophy - 80%

How well did you know this?
1
Not at all
2
3
4
5
Perfectly
13
Q

What injury pattern has the lowest rate of ligament injury in a tibial plateau fracture?

A

lateral split

How well did you know this?
1
Not at all
2
3
4
5
Perfectly
14
Q

Which dwarfism does not have an ustable c-spine?

A

achondroplasia

How well did you know this?
1
Not at all
2
3
4
5
Perfectly
15
Q

What is the most common cause of heel pain in the pediatric population?

A

Sever’s

How well did you know this?
1
Not at all
2
3
4
5
Perfectly
16
Q

Acceptable alignment of a humeral shaft fracture

A

<!--StartFragment-->

< 20° anterior angulation
< 30° varus/valgus angulation
< 3 cm shortening

<!--EndFragment-->

How well did you know this?
1
Not at all
2
3
4
5
Perfectly
17
Q

What is true about the genetics and etiology of achondroplasia?

A
  • FGFR3 point mutation G380R
    • AD
    • 80% are spontaneous
      • associated with higher paternal age
  • Leads to abnormal chondroid production in the proliferative zone
    • short long bones
    • endosteal ossification
    • proximal > distal (?why)
  • Spine
    • narrow interpedicular distance (L1-5)
    • narrow foramen magnum
      • mortality, apnea, UMN
      • screen all with MRI who have positive sleep study
    • thoracolumbar kyphosis
      • brace when young
      • usually will resolve
    • lumbosarcral hyperlordosis
    • spinal stensosis
      • narrow pedicles
      • hyperplastic discs
  • MSK
    • genu varum
    • rhizomelic shortening
    • trident hand
How well did you know this?
1
Not at all
2
3
4
5
Perfectly
18
Q

What structures are released during plantar fascial release

A
  • divide supericial fascia of the abductor hallucis
  • fascia between abductor hallucis and plantar quadratus is released
    • releases the nerve between the two
  • a portion of the plantar fascia is incised
  • medial heel spur is resected if present
How well did you know this?
1
Not at all
2
3
4
5
Perfectly
19
Q

Treatment of unstable PIP fracture-dislocations

A
  • Extension block pinning (transarticular)
    • reducible joint with tenuous stability
    • flex the joint slightly and pin the proximal phalax
  • CRPP
    • reducible joint with tenous stability
    • larger fragment ammenable to pinning
    • not superior to extension pinning
  • Dynamic distraction and Ex-fix
    • unstable pilon or > 50% (B type fracture)
    • off-loads the joint and permits early ROM
  • Volar Plate arthroplasty
    • augmented with button
    • proected with k-wire
  • Hemihamate reconstruction arthroplasty (more for volar fractures)
    • section of hamate if measured and transplated
    • B type fractures can use hemiH or dynamic ex-fix; no studies to compare the two
    • 2 week extension splint then ROMAT
    • no long term studies
    • easier to do in a more chronic situation where you can’t do a DynEx
How well did you know this?
1
Not at all
2
3
4
5
Perfectly
20
Q

What is the most common complication of a talar neck fracture?

A

subtalar OA

How well did you know this?
1
Not at all
2
3
4
5
Perfectly
21
Q

What are the indications for epiphyseodesis in congential scoliosis

A
  • Age <5 years
  • Site of anomaly in lumbar as opposed to thoracic spine
  • A curve of five segments or less
  • A progressive curve <40°
  • Anomalies consisting of hemivertebrae instead of unsegmented bars
  • Absence of excessive kyphosis
  • Absence of neurological deficits, including syrinx, diastomatomyelia and tethered spinal cord
How well did you know this?
1
Not at all
2
3
4
5
Perfectly
22
Q

What are the MRI characteristics and treatment of an ABC?

A
  • MRI - cystic, fluid-fluid levels showing blood and solid components
  • curettage and grafting with cryotherapy
How well did you know this?
1
Not at all
2
3
4
5
Perfectly
23
Q

Approach to adolescent bunions

A
  • best to wait until skeletal maturity to operate
  • can not perform metatarsal osteotomies if physis is open (cuneiform osteotomy OK)
  • surgery indicated in symptomatic patients with an IMA > 10° and HVA of > 20°
  • severe deformity with a DMAA > 20 perform a double MT osteotomy
  • technique
    • soft tissue procedure alone not successful
    • similar to adults if physis is closed (except in severe deformity)
How well did you know this?
1
Not at all
2
3
4
5
Perfectly
24
Q

What is the most sensitive test to detect OM?

A

MRI

How well did you know this?
1
Not at all
2
3
4
5
Perfectly
25
Q

What is the best treatment for a 20 yo female with JRA and trashed TN, CC and TC joints?

A

triple arthrodesis

How well did you know this?
1
Not at all
2
3
4
5
Perfectly
26
Q

What is the nonopertive treatment of spondylolisthesis?

A

Asx - non-op, full contact

Sx low grade - PT for pelvic tilt, core strength and hamstring stretches (know the risks and monitor)

Acute - TLSO, better than activity mod alone

How well did you know this?
1
Not at all
2
3
4
5
Perfectly
27
Q

What are the sensitivity/specificity/PPV/NPV

A
  • sensitivity = (true positive) / (true positive + false negative)
    • proportion of individuals who truly have a designated disorder who are so identified by test
  • specificity = (true negative) / (true negative + false positive)
    • proportion of individuals who are truly free of a designated disorder who are so identified by test
  • PPV = (True positive) / (True + False positives)
    • proportion of individuals with a positive test who have the disease
  • NPV = (true negative) / (true + false negatives)
    • proportion of individuals with a negative test who are free of the disease
How well did you know this?
1
Not at all
2
3
4
5
Perfectly
28
Q

What is the typical presentation of a glomus tumor

A
  • Small red-blud nodule on gross histoloy
  • typically in the nailbed
    • can occur in other places
    • riding or discoloration of the nailbed
  • 20-40 yo
  • F>M in subungual region only
  • Triad
    • paroxysmal pain
    • cold intolerance
    • localized tenderness
  • XR
    • may not be helpful
    • scalloped osteolystic defect with sclerotic border
  • MRI
    • diagnostic
How well did you know this?
1
Not at all
2
3
4
5
Perfectly
29
Q

What are indications for surgery of a clavicle fracture?

A

unstable Group II fxs (Type IIA, Type IIB, Type V)

open fxs
displaced fracture with skin tenting
subclavian artery or vein injury
floating shoulder (clavicle and scapula neck fx)
symptomatic nonunion
posteriorly displaced Group III fxs

displaced Group I (middle third) with >2cm shortening

How well did you know this?
1
Not at all
2
3
4
5
Perfectly
30
Q

What is the effect of overstuffing a radial head arthroplasty

A
  • increased load
  • limit mobility
How well did you know this?
1
Not at all
2
3
4
5
Perfectly
31
Q

Anatomy of the blood supply to the hand

A
  • Superficial arch
    • deep to palmar fascia
    • distal to the to the deep arch
    • at the level of a line drawn across the palm parallel to the distal edge of the fully abducted thumb
    • predominant supply is ulnar artery
      • 1st branch
        • is the deep branch that provides the minor supply to the deep palmar arch
      • 2nd branch
        • is the ulnar digital artery of the little finger
      • 3rd, 4th, 5th, and 6th branches
        • are the common palmar digital arteries
  • Location compared to nerves
    • in the palm, the digital arteries are volar to the digital nerves
    • in the digits, the digital arteries are dorsal to the digital nerves
    • in the digits, the neurovascular bundle is volar to Cleland’s ligament (ceiling, grayson = ground)
  • Deep Arach
    • deep to the flexor tendons (FDS, FDP)
    • proximal to the superficial arch
    • at the level of the base of the metacarpals
    • predominant supply is the deep branch of the radial artery
      • princeps pollicis
        • runs between 1st dorsal interosseus and adductor pollicis
      • branch to the radial side of the index finger
        • the proper digital artery to the radial side of the IF arises directly from the deep arch
      • branches to the 3 common digital arteries in the 2nd, 3rd, and 4th web spaces
    • The deep arch is complete (branches to all digits) in 97% of individuals
How well did you know this?
1
Not at all
2
3
4
5
Perfectly
32
Q

What are the 5 mechanisms of wear in TJA

A
  • Adhesive wear
    • most important in osteolytic process
    • microscopically PE sticks to prosthesis and debris gets pulled off
  • abrasive wear
    • cheese grater effect of prosthesis scraping off particles
  • third body wear
    • particles in joint space cause abrasion and wear
  • volumetric wear
    • main determinant of number of particles created
    • directly related to square of the radius of the head
    • volumetric wear more or less creates a cylinder
    • V=3.14rsquaredw
    • V is volumetric wear, r is the radius of head, w is linear head wear
    • head size is most important factor in predicting particles generated
  • Linear wear
    • is measured by the distance the prosthesis has penetrated into the liner
How well did you know this?
1
Not at all
2
3
4
5
Perfectly
33
Q

What is the treatment for the different stages of PTTI?

A

Grade 1 - debridment and tendon augmentation

Grade 2 - FDL transfer, medial calcaneal osteotomy, TAL, +/-cotton, +/-evans

Grade 3 - subtalar arthrodesis

Grade 4 - Subtalar arthrodesis with deltoid recon if ankle is flexible

TTC if ankle valgus is fixed

How well did you know this?
1
Not at all
2
3
4
5
Perfectly
34
Q

Most common presentation of discitis in peds

A

> 9 back pain

3-9 abdo pain**

< 3 limp, posivite log roll

How well did you know this?
1
Not at all
2
3
4
5
Perfectly
35
Q

What is ture about NOF?

A

cortical

metaphyseal/diaphyseal

1-3cm

bubbly - not loculated

Eccentric

How well did you know this?
1
Not at all
2
3
4
5
Perfectly
36
Q

What is Morquio syndrome?

A

Mucopolysacharidosis

accumulation of keratn sulfate

presents at 18-24 months, proportionate dwarfism

normal intellegence, waddling gait, genu valgum, thoracic kyphosis, corneal clouding

XR - thick skull, wide ribs, vertebral beaking, coxa vera, unossified femoral head

*** Flex - ex views for C1-C2 instability

How well did you know this?
1
Not at all
2
3
4
5
Perfectly
37
Q

Complications associated with CTR

A
  • inadequate release (most common)
  • motor branch disection
  • hypertrophic scar
  • pillar pain - pain over trapeizium and hamate prominence
    • can result from division of the palmar cuteanous branch
    • some sources quote this as more common
  • adhesions
  • laceration superificial palmar arch
  • hematoma
  • recurrence
How well did you know this?
1
Not at all
2
3
4
5
Perfectly
38
Q

What is the treatment for chronic achilles tear?

A

Type 1: gap < 2cm = end-to-end repair
Type 2: gap 2 to 5 cm = V-Y lengthening +/- tendon transfer
Type 3: gap >5cm = (FHL) tendon transfer alone +/- V-Y advancement

How well did you know this?
1
Not at all
2
3
4
5
Perfectly
39
Q

What are the common causes of wear in a metal on poly hip?

A

Mode 1 - bearing surface (adhesive, abrasive; MOST COMMON)

Mode 2 - bear on non-bearing; neck on poly

Mode 3 - 3rd body wear (poly, debris)

Mode 4 - non-bearing with non-bearing; backside wear (trunionosis)

How well did you know this?
1
Not at all
2
3
4
5
Perfectly
40
Q

What are the primary and secondary lesions associated with swan neck deformity

A
  • Primary
    • trauma
    • generalized ligament laxity
    • rheumatoid arthritis
  • Secondary
    • mallet injury
      • leads to transfer of DIP extension force into PIP extension forcee
  • FDS rupture
    • leads to unopposed PIP extension with an intact FDP
  • intrinsic contracture
    • tethering of the lateral (collateral) bands by the transverse retinacular ligament as a result of PIP hyperextension.
    • if the lateral (collateral) bands are tethered, excursion is restricted and the extension force is not transmitted to the terminal tendon, and is instead transmitted to the PIP joint
  • MCP joint volar subluxation
    • caused by rhuematoid arthritis
How well did you know this?
1
Not at all
2
3
4
5
Perfectly
41
Q

Brachial plexus palsy: preganglionic vs postganglionic

A
  • preganglionic
    • avulsion proximal to dorsal root ganglion
    • involves CNS which does not regenerate – little potential recovery of motor function (poor prognosis)
    • lesions suggesting preganglionic injury:
      • Horner’s syndrome
        • disruption of sympathetic chain
      • winged scapula medially
        • loss of serratus anterior (long thoracic nerve),rhomboids (dorsal scapular nerve)
      • presents with motor deficits (flail arm)
      • sensory deficits
      • absence of a Tinel sign or tenderness to percussion in the neck
      • normal histamine test (C8-T1 sympathetic ganglion)
        • intact triple response (redness, wheal, flare)
      • elevated hemidiaphragm (phrenic nerve
      • rhomboid paralysis (dorsal scapular nerve)
      • supraspinatus/infraspinatus (suprascapular nerve)
      • latissimus dorsi (thoracodorsal)
      • evaluation
        • EMG may show loss of innervation to cervical paraspinals
  • postganglionic
    • involve PNS, capable of regeneration (better prognosis)
    • presentation
      • presents with motor deficit (flail arm)
      • sensory deficits
    • evaluation
      • EMG shows maintained innervation to cervical paraspinals
      • abnormal histamine test
        • only redness and wheal, but NO flare
How well did you know this?
1
Not at all
2
3
4
5
Perfectly
42
Q

What is true about the use of BMP-2 in open tibia fractures?

A

BEST

decreased need secondary intervention or subsequant bone grafting

increased healing rate

decreased infection in grade III only

How well did you know this?
1
Not at all
2
3
4
5
Perfectly
43
Q

Why do you not get lunate AVN with volar dislocation?

A

radioscaphoid ligament - ligament of testut is intact in all 4 stages of dislocation

How well did you know this?
1
Not at all
2
3
4
5
Perfectly
44
Q

What is true about anterior subcutaneous transposition and simple decompression for the ulnar nerve?

A

same functional outcome with higher infection rates with transposition (conchrane review from 2012)

How well did you know this?
1
Not at all
2
3
4
5
Perfectly
45
Q

Intrinsic tendon thumb insertions

A
How well did you know this?
1
Not at all
2
3
4
5
Perfectly
46
Q

What is the most common reason for sudden death in an RA patient?

A

basilar invagination

How well did you know this?
1
Not at all
2
3
4
5
Perfectly
47
Q

what is the treatment for schermans kyphosis

A

< 60 - observe

60-80 and risser < 3 - milwakee brace

>75 in skeletally mature - PSF

How well did you know this?
1
Not at all
2
3
4
5
Perfectly
48
Q

What additional tests and consults are required for congential scoliosis

A
  • Cardiac
    • cardiology consult
    • echocardiogram
  • Renal: common to have kidney issues
    • renal ultrasound (at minimum)
  • Neurologic
    • spine MRI
      • syrinx
      • tethering
    • neurosurg referral
How well did you know this?
1
Not at all
2
3
4
5
Perfectly
49
Q

What are the surgical priorities of tendon transfers?

A

<!--StartFragment-->

  • elbow flexion (musculocutaneous n.)
  • shoulder stabilization (suprascapular n.)
  • brachiothoracic pinch (pectoral n.)
  • sensation C6-7 (lateral cord)
  • wrist extension and finger flexion (lateral and posterior cords)

<!--EndFragment-->

How well did you know this?
1
Not at all
2
3
4
5
Perfectly
50
Q

What is the most common level of hip dislocation in myelomeningocele?

A

L3

How well did you know this?
1
Not at all
2
3
4
5
Perfectly
51
Q

What is an important consideration for myelomeningocele?

A

Latex allergy

How well did you know this?
1
Not at all
2
3
4
5
Perfectly
52
Q

What is the true regarding the treatment of OCD of the knee

A
  • Stable lesions in a skeletally immature patient should not be treated surgically
  • Unstable lesions should be treated
    • reduce pain
    • reduce OA - this is only theoretical
  • Should be give PT
  • Nothing to show that OATS or ACI are any better than microfracture
    • microfracture should be first line of treatment
  • Outcomes of microfracture
    • Age
    • Duration of symptoms
    • Lesion size
    • Body mass index
    • Preoperative activity level >4 (Tegner)
    • Previous surgery (microfracture)
    • Repair cartilage volume >66%
      • MRI results of defect fill
How well did you know this?
1
Not at all
2
3
4
5
Perfectly
53
Q

What is your management for an anterior SC dislocation?

A
  • > 3 weeks = leave it
  • < 3 weeks
    • closed reduction under anesthesia
    • figure 8 brace if acceptable
    • leave it if unable to reduce
  • chronic and painful
    • medial clavicle resection
How well did you know this?
1
Not at all
2
3
4
5
Perfectly
54
Q

What is the most important determining factor in AVN associated with pedicatric femoral neck fracture?

A

Age 3-8 (after 8 have the ligamentum teres; after 14 have metaphyseal flow)

  • Dalbet classification
    • Type I
      • Transphyseal (IA, without dislocation of epiphysis from acetabulum; IB, with dislocation of epiphysis)
      • 90-100% AVN
    • Type II
      • Transcervical 40-50%
      • 50% AVN
      • 15% nonunion
    • Type III
      • Cervicotrochanteric (or basicervical) 30-35%
      • 25% AVN
      • 15-20% nonunion
    • Type IV
      • Intertrochanteric
      • 10% ANV
      • 5% Nonunion
How well did you know this?
1
Not at all
2
3
4
5
Perfectly
55
Q

What are the common principles for tendon transfers?

A
  • donor must be expendable and of similar excursion and power
  • one tendon transfer performs one function
  • synergistic transfers rehabilitate more easily
  • it is optimal to have a straight line of pull
  • one grade of motor strength is lost following transfer
  • only cross one joint
  • Other concepts
    • appropriate excursion
      • <!--StartFragment-->

3 cm excursion - wrist flexors, wrist extensors
5 cm excursion - EDC, FPL, EPL
7 cm excursion - FDS, FDP

* appropriate tensioning

<!--EndFragment-->

How well did you know this?
1
Not at all
2
3
4
5
Perfectly
56
Q

What is true about the pathology of RA

A
  • RF is an IgM antibody associated with RA a number of other autoimmune disorders
    • attackes IgG
    • monoclonal cells are the primary mediator of tissue destruction
      • present in 1-2% of the general population
  • prognostic value more than diagnostic value
  • Diagnositc criteria
    • morning stiffness and swelling
    • subcutaneous nodules
    • positive laboratory tests
      • anti-CCP is the most sensitive
    • radiographic findings
How well did you know this?
1
Not at all
2
3
4
5
Perfectly
57
Q

What are indications for total patellectomy

A
  • failed ORIF
  • infection
  • tumor
  • PF OA
  • 47% of the strength of the quads is lost
  • should reinforce with VMO over the defect (shown to have better outcomes)
How well did you know this?
1
Not at all
2
3
4
5
Perfectly
58
Q

What are indications to perform corrective valgus osteotomy for coxa vara?

A
  1. Hilgenreiner’s physeal angle > 60
    • ​​<45 unlikely to progress
    • >60 will likely progress
  2. Neck-shaft angle < 110
  3. trendelenberg
  4. pain
  5. evidence of progression with serial exams

Important considerations - may need to transfer aductor tubercle to increase tension, monitor for LLD

How well did you know this?
1
Not at all
2
3
4
5
Perfectly
59
Q

What is the pathoanatomy of a SLAC wrist?

A
  • chronic SL ligament injury creates a DISI deformity
  • scaphoid is flexed and lunate is extended as scapholunate ligament no longer restrains this articulation
  • scapholunate angle > 70 degrees
  • lunate extended > 10 degrees past neutral
  • Resulting forces
    • abnormal distribution of forces across midcarpal and radiocarpal joints
    • malalignment of concentric joint surfaces
    • initially affects the radioscaphoid joint and progresses to capitolunate joint
How well did you know this?
1
Not at all
2
3
4
5
Perfectly
60
Q

What is the incidence of neuraxial abnormalities with juvinille scoliosis?

A
  • Children 3-10 yo
  • high incidence of neural axis abnormalities (18-25%)
    • syringomyelia
      • cyst or tubular cavity within spinal cord (due to increase CSF from tumor, arnold chiari..etc, need to drain it and treat underlying cause)
      • can be seen in a scoliotic curve without rotation
      • can manifest as an asymmetric umbilicus reflex
    • Arnold-Chiari syndrome
      • cerebellar tonsil are elongated and protruding through the opening of the base of the skull and blocking CSF flow)
    • tethered cord
    • dysraphism
    • spinal cord tumor
How well did you know this?
1
Not at all
2
3
4
5
Perfectly
61
Q

What is the enneking tumour grading system?

A
How well did you know this?
1
Not at all
2
3
4
5
Perfectly
62
Q

Principles of medical ethics

A
  • Respect for autonomy - the patient has the right to refuse or choose their treatment
  • Beneficence - a practitioner should act in the best interest of the patient
  • Non-maleficence - “first, do no harm”
  • Justice - concerns the distribution of scarce health resources, and the decision of who gets what treatment (fairness and equality)
  • Respect for persons - the patient (and the person treating the patient) have the right to be treated with dignity.
  • Truthfulness and honesty - the concept of informed consent has increased in importance since the historical events of the Doctors’ Trial of the Nuremberg trials and Tuskegee syphilis experiment.
How well did you know this?
1
Not at all
2
3
4
5
Perfectly
63
Q

What are the features of Marfan?

A

Fibrillin Gene

Ortho - arachnodactyly, scoliosis, protrusio, laxity, dislocations, pes planovalgus

Non-ortho - aortic root dilations, aortic disection, mitral valve prolapse, pectus excavatum, pneumothorax, dural exctasia, meningocele

How well did you know this?
1
Not at all
2
3
4
5
Perfectly
64
Q

What are the sites of compression of the ulnar nerve in the cubital tunnel?

A

*Arcade of struthers

*Aponeurosis of FCU

*MCL and osborne’s ligament

medial head of triceps

intermuscular septum

medial epicondyle

anconeus epitrochleris

How well did you know this?
1
Not at all
2
3
4
5
Perfectly
65
Q

What is the watson classification for SLAC wrist?

A
  • Stage I SLAC wrist - scaphoid and syloid
    • PA radiograph shows radial styloid beaking, sclerosis and joint space narrowing between scaphoid and radial styloid
  • Stage II SLAC wrist - scaphoid and entire DR
    • PA radiograph shows sclerosis and joint space narrowing between scaphoid and the entire scaphoid fossa of distal radius
  • Stage III SLAC wrist - capitate and lunate
    • PA radiograph shows sclerosis and joint space narrowing between the lunate and capitate, and the capitate will eventually migrate proximally into the space created by the scapholunate dissociation
How well did you know this?
1
Not at all
2
3
4
5
Perfectly
66
Q

What is the best test to examine osteoid osteoma?

A

CT - better than MRI to look for the nidus

How well did you know this?
1
Not at all
2
3
4
5
Perfectly
67
Q

What are the risk factors of hip AVN?

A
How well did you know this?
1
Not at all
2
3
4
5
Perfectly
68
Q

Diagnosis? Blocks to reduction? Treatment?

A
  • Dorsal IP dislocation
  • late deformity
    • swan neck - due to volar plate injury
    • flexion deformity - resolves with PT
  • Blocks to reduction
    • ​volar plate
    • If open injury - FDP dislocation
  • ​Treatment
    • ​buddy tape for 3-6 weeks if reducible
    • open to get reduction
      • ​dorsal approach between central slip and lateral bands
How well did you know this?
1
Not at all
2
3
4
5
Perfectly
69
Q

Diagnosis? Imaging? Treatment?

A

Pediatric Odontoid fracture

  • Mechanism of injury
    • high velocity flexion
    • anterior displacement through the synchondrosis
  • Acutely
    • Usally seen on lateral
    • Can get CT to assess
    • Flex-ex views if appropriate
    • MRI - static if chronic to assess cord
      • Then can do dynamic MRI to assess stability of the fragment
  • Treatment
    • Reduction with Halo or Minerva for 3 months
      • Acute
      • Confirm reduction with XR
      • Confirm healing with fle-ex views
    • Usually heals with NO issues
  • Posterior C1/2 fusion (chronic)
    • If no decompression necessary
    • Brooks/Gallie - if young
    • C1/2 lateral mass screws if older
  • Posterior Occiput-C2 fusion (unstable os odontoidium)
    • For chornic injuries that you need to decompress posteriorly
    • Otherwise hard to get your bone graft in
How well did you know this?
1
Not at all
2
3
4
5
Perfectly
70
Q

Cross-link in pedicle screw construct improves what stiffness?

A
  • lateral Bending
  • then torsion
How well did you know this?
1
Not at all
2
3
4
5
Perfectly
71
Q

What is this pathology? What is the appropriate treatment?

A
  • Vaughan-Jackson syndrome (RA)
    • describes the rupture of the hand digital extensor tendons which occur from the ulnar side of the wrist first then moves radially
  • pathoanatomy
    • results when DRUJ instability results in dorsal prominence of the ulnar head which results in an attritional rupture of the extensor tendons
    • EDM is the first extensor ruptured.
  • treatment
    • EIP to EDC transfer and distal ulna resection
How well did you know this?
1
Not at all
2
3
4
5
Perfectly
72
Q

What are the prognostic factors associated with OCD?

A
  • Age - younger age better prognosis
  • Lesion size less than 4cm2 show better prognosis
  • Activity level- High impact athletes have worse prognosis though still improved
  • Increased body weight- worse prognosis
  • Patello-femoral lesions worse
  • Better fill of defect on MRI correlated to better clinical results
How well did you know this?
1
Not at all
2
3
4
5
Perfectly
73
Q

What is the diagnosis and treatment for nec fasc?

A
  • pain out of proportion
  • systemic
  • bullae
  • Fascial edema from infection occludes arterial flow
    • promotes spread of infection
    • dishwater pus
    • Nectrotic skin follows
  • Most commonly GAS
    • diabetic, HIV and malignancy at higher risk (but 50% in healthy patients)
How well did you know this?
1
Not at all
2
3
4
5
Perfectly
74
Q

What is often associated with CVT?

A

myelomeningocele

tethered cord

arthrogryposis

How well did you know this?
1
Not at all
2
3
4
5
Perfectly
75
Q

What is the measurement of the Kaplan - Meier survivorship curve?

A

number failed divided by the number still being followed

How well did you know this?
1
Not at all
2
3
4
5
Perfectly
76
Q

Medical treatment of spasticity in CP

A
  • Benzodiazepine
    • works systemically to reduce spasms and anxeity
    • used with caution due to effects of sedation
  • Bacolfen
    • works centrally to decrease GABA receptors
    • works to inhibit Ca influx from pre-synapic vessels
    • drug of choice for spinal cord pathology in adults
  • Dantrolene
    • works peripherally on the muscles to decrease the release of Ca from the sarcoplasmic reticulum
  • Botox - not given centrally
    • injected into the muscle
    • works locally to block release of Ach from pre-synaptic vessels
  • Tizanidine - new with less sedation than some of the older medications
How well did you know this?
1
Not at all
2
3
4
5
Perfectly
77
Q

What is true about the stability of SI screws

A
  • 2 screws have higher load to failure
  • anterior plate with 2 screws is not as stable as native pelvis
How well did you know this?
1
Not at all
2
3
4
5
Perfectly
78
Q

What are the risk factors of progression for a congential scoliosis?

A

Rib Phase 1 (>2 will progress)

RVAD/Mheta < 20 (90% will resolve spont)

Cobb < 25

Double major curves progress

How well did you know this?
1
Not at all
2
3
4
5
Perfectly
79
Q

What artery supplies the lateral arm flap

A

posterior radial collateral artery

How well did you know this?
1
Not at all
2
3
4
5
Perfectly
80
Q

What position should you fuse the ankle?

A

neutral dorsiflexion

5 valgus

10 ER

How well did you know this?
1
Not at all
2
3
4
5
Perfectly
81
Q

Charnley cemented THA in 40-45 yo, survival at 10 years?

A

89%

How well did you know this?
1
Not at all
2
3
4
5
Perfectly
82
Q

Boney landmarks for the anterior cervical spine

A

Hard palate-arch of the atlas

Lower border of the mandible-C2-3

Hyoid bone-C3

Thyroid cartilage-C4-5

Cricoid cartilage-C6

Carotid tubercle-C6

How well did you know this?
1
Not at all
2
3
4
5
Perfectly
83
Q

What is the mechanism of botox?

A

blocks ACTH from release at the pre-synapic vesicles

How well did you know this?
1
Not at all
2
3
4
5
Perfectly
84
Q

What are the charactersitics of proteus syndrome

A
  • skin overgrowth and boney deformation
  • genetic mosacism
    • often get overgrowth (hemihypertrophy)
    • megalospondylodysplasia
  • at risk of developing tumors
How well did you know this?
1
Not at all
2
3
4
5
Perfectly
85
Q

What is the nonoperative indication for pec major tears?

A

Partial ruptures

Proximal tears ( in the muscle or musculotendonous junction)

Sedentary patient

How well did you know this?
1
Not at all
2
3
4
5
Perfectly
86
Q

What is the treatment algorythm for a bennet fracture?

A

<1mm = cast

> 1mm and large = screw

> 1mm and small = k-wire (wagner pinning)

(one=oblique ligament, one=counter tendons)

How well did you know this?
1
Not at all
2
3
4
5
Perfectly
87
Q

What is the most common complication of a monteggia fracture?

A

PIN

Usually will resolve will closed reduction

Chronic nerve palsy is very rare, pateients complain of weakened power grip due to inability to extend wrist (can treat initially with PT to maintain ROM and then with tendon transfers)

How well did you know this?
1
Not at all
2
3
4
5
Perfectly
88
Q

Presentation and treatment of ACL ganglion cyst

A
  • Presents with pain with flexion, especially deep flexion
    • after activity
    • mechanical symptoms
  • Able to make diagnosis from MRI
  • Classification
    • anterior to ACL
    • between ACL and PCL
    • posteior to PCL
  • Treatment
    • arthroscopic vs image guided aspiration
    • both have good outcomes with limited reported recurrence
How well did you know this?
1
Not at all
2
3
4
5
Perfectly
89
Q

What are the characteristics of Pasteurella

A
  • genus of Gram-negative, aerobic but facultatively anaerobic bacteria
  • occur in domestic pet bites (dog and cat)
  • it is not difficult to isolate
  • treatment
    • tetanus, I&D
    • clavulin, cipro/flouroquinolone, septra
How well did you know this?
1
Not at all
2
3
4
5
Perfectly
90
Q

In a hypoplastic thumb, what is an indication for pollicization of the 1st digit with thumb amputation?

A

unstable CMC joint

How well did you know this?
1
Not at all
2
3
4
5
Perfectly
91
Q

What are the important considerations when treating an adult brachial plexus injury?

A

Patient Selection

Timing of surgery

Prioritization of restoration

How well did you know this?
1
Not at all
2
3
4
5
Perfectly
92
Q

What is the Neer-Holwitz classification for pediatric proximal humerus fracture?

A

<!--StartFragment-->

type I : < 5mm
type II : < 1/3 of the shaft width
type III : > 1/3 and < 2/3 the shaft width
type IV : > 2/3 the shaft width

<!--EndFragment-->

How well did you know this?
1
Not at all
2
3
4
5
Perfectly
93
Q

What are the treatment options for SLAC wrist by stage?

A
  • Stage 1 - radial syloidectomy and scaphoid stabilitzation
  • Stage 2 - proximal row carpectomry
    • ​CI if there is severe capitate OA
  • ​Stage 3 - four corner fusion
    • ​no evidence to show superior to proximal row carpectomy
  • Wrist Fusion
    • option for pancarl OA
    • good pain control and return of grip with loss of motion
How well did you know this?
1
Not at all
2
3
4
5
Perfectly
94
Q

What are the indications and contraindications for doing a TLIF?

A
  • Indications
    • spondylolisthesis (low grade)
    • degenerative scoliosis
    • severe instability
    • pseudarthrosis
    • recurrent disk herniation
    • painful degenerative disk disease
  • Contraindications
    • epidural fibrosis
    • active infection
    • conjoined nerve roots
      • restrict access to the disk space
    • severe disk space collapse
    • severe ankylosis
    • osteoporosis
How well did you know this?
1
Not at all
2
3
4
5
Perfectly
95
Q

What is the classification of muchopolysaccaroidosis

A
  • Hurler
    • most severe
    • α-L-iduronidase
    • Dermatan sulfate, heparan sulfate
  • Hunter
    • Iduronate-2-sulfatase
    • Dermatan sulfate, heparan sulfate
  • San Filipino
    • most common
    • Heparan sulfate, chondroitin sulfate
  • Morqouio
    • Keratan sulfate, chondroitin sulfate
How well did you know this?
1
Not at all
2
3
4
5
Perfectly
96
Q

What is the best way to access cross sectional invovlement of tarsal coalition?

A

CT scan

How well did you know this?
1
Not at all
2
3
4
5
Perfectly
97
Q

What are the common complications associated with ankle arthroscopy?

A

<!--StartFragment-->

  • Synovial cutaneous fistula
    • avoid by immobilization to allow portal skin healing and closure
  • Neurovascular injury from portal placement
  • Anterolateral portal
    • risks dorsal intermediate cutaneous branch of superficial peroneal nerve
    • most common neurovascular injury
  • Anteromedial portal
    • risks saphenous nerve and vein
  • Anterocentral portal
    • risks dorsalis pedis artery
  • Posterolateral portal
    • risks sural nerve and small saphenous vein
  • Posteromedial portal
    • risks posterior tibial artery

<!--EndFragment-->

How well did you know this?
1
Not at all
2
3
4
5
Perfectly
98
Q

Compare osteoporosis and osteomalaceia

A
  • Osteoporosis
    • reduced bone mass
    • Type 1 - postmenopausal
      • trabecular
      • distal radius, VB (and IT hip)
      • reduced Vit D and intake
    • Type 2 - elderly
      • trabecular>cortical
      • hip/pelvis (femoral neck)
      • reduced Ca intake
    • pain at fracture
    • Labs
      • Ca, ALP and urinary Ca are normal
    • Tetracycline labelling - normal
    • Osetopenia - 1-2.5 SD below 25 yo
    • Osteoporosis - >2.5 SD below 25yo
  • Osteomalacia
    • reduce bone mineralization
    • generalized bone pain
    • appedicular fractures, deformity and looser zones
    • Labs
      • will be altered based on etiology (rickets, hyperparathyroid, hypophosphatasia)
    • abnormal tetracycline labelling
How well did you know this?
1
Not at all
2
3
4
5
Perfectly
99
Q

Use of aprotinin to reduce blood loss in hip and spine surgery

A

Trasylol, or aprotinin; Historic drug used to prevent blood loss by inhibiting fibrinolysis. Was taken off the market in 2008 due to complications including death

How well did you know this?
1
Not at all
2
3
4
5
Perfectly
100
Q

What are the important techniques when applying halo?

A
  • Movement Restriction
    • More motion below C2
    • Controversial - most motion at C2/3 and least at C7/T1
    • Some say allows 70% of flexion
  • Contraindications
    • absolute
      • cranial fracture
      • infection
      • severe soft-tissue injury at the proposed pin sites
    • relative
      • severe chest trauma
      • obesity, advanced age
      • recent evidence demonstrates an unacceptably high mortality rate in patients aged 79 years and older (21%)
      • barrel-shaped chest
  • Indications for CT
    • possible fracture
    • child
  • Adults
    • total of 4 pins
      • 2 posterior pins
        • 1 cm above the pinna
      • 2 anterior pins
        • lateral 1/3 of the eyebrow
        • this is anterior/medial to temporalis fossa
        • this is lateral to supraorbital nerve
    • tighten to 4-8 inch-pounds of torque
      • (not foot pounds..)
    • have patient return on day 2 to tighten again
  • Pediatrics
    • more pins, less torque
    • total of 6-8 pins
    • lower torque (2-4 in-lbs or “finger-tight”)
      *
How well did you know this?
1
Not at all
2
3
4
5
Perfectly
101
Q

What is the most common nerve injury in THA? What is the recovery?

A
  • Sciatic (Peroneal division)
  • most often damaged by retractors
    • 70% of people will have subclinical findings
  • Risk factors
    • Women
    • DDH
    • Revsion surgery
  • Do to the large size of the nerve and the proximal from the end organ recovery is rare
  • Treatment is supportive
  • Prevention is the best stragtegy
How well did you know this?
1
Not at all
2
3
4
5
Perfectly
102
Q

What is the harrington technique described for acetabular deficit

A

antiprotrusio cage with screws

liner secured PMMA

3 rods to reconstruct posterior column of the acetabulum

How well did you know this?
1
Not at all
2
3
4
5
Perfectly
103
Q

If you have a normal ESR/CRP do you need to get further imaging?

A

No

How well did you know this?
1
Not at all
2
3
4
5
Perfectly
104
Q

What is the vancouver classification and associated treatments

A
  • A; L (lesser), G (greater)
    • Occurs in trochanteric region
    • Commonly associated with osteolysis and often requires treatment that addresses the osteolysis
    • Often will require ORIF with claw plate if adequate bone stalk
  • B1
    • Fx around stem or just below it with a well fixed stem
    • Often unreconized B2 - make sure you assess the XR adequatly
    • ORIF using cerclage cables and locking plates
  • B2
    • Fx around stem or just below it, loose stem, but good proximal bone stock
    • Revision of the femoral component to a long porous-coated cementless stems (fluted, tapered)and fixation of the fracture fragment. Revision of the acetabular component if indicated.
    • Bypass # - 1.5cm
    • May need ETO to get the component out, fix with circlage wires
  • B3
    • Fx around stem or just below with proximal bone that is poor quality or severely comminuted
    • Femoral component revision with proximal femoral allograftn (young) or proximal femoral replacement (old and low demand)
    • Consider the demand of your patient - femoral replacement only for low demand patients
  • C
    • Fx occurs well below the prosthesis
    • ORIF and with plate
How well did you know this?
1
Not at all
2
3
4
5
Perfectly
105
Q

What is the advantage of lateral column lengthening over medial calcaneal slide?

A

better forefoot aduction

How well did you know this?
1
Not at all
2
3
4
5
Perfectly
106
Q

Compare observational and experimental designs for research studies

A
  • observational
    • researchers observe patient groups without allocation of intervention
    • may be either prospective or retrospective
    • may be descriptive or analytic
      • descriptive
        • useful for obtaining background information for more advance studies
        • examples
          • case reports
          • case series
          • cross-sectional studies
      • analytic
        • explores the association between a given outcome and a potentially related variable
        • examples
          • case-control
          • cohort
          • meta-analysis
  • experimental
    • researchers allocate treatment
    • allows the evaluation of efficacies of therapeutic interventions
    • examples
      • double-blinded, prospective, randomized clinical trial is the gold standard for evidence based medicine
How well did you know this?
1
Not at all
2
3
4
5
Perfectly
107
Q

What is true about safe tourniqutte use?

A
  • For a given leg circumfrance, a wider cuff means a lower pressure
  • ischmeic injury to nerves can occur, thought that muscles are more sensitive than nerves
    • > 2 hrs
    • 350mmhg - lower
    • 250 mmhg - upper
  • EMG findings in 70% of patients 5 months later
    • slower recovery
  • muscles can recover from periodic ‘rest’ during surgery but nerves do not
  • Consider the SBP to use the smallest measured pressure
  • How to reduce tourniquette damage
    • wider cuff
    • cool extremity
    • 2 cuffs, alternate use
    • pad the extremity
    • don’t use prep
How well did you know this?
1
Not at all
2
3
4
5
Perfectly
108
Q

Most important prognositc factor surrounding this fracture?

A

PIP subluxation inhibits the gliding arc of the joint and portends a poor clinical outcome (therefore, congruent joint reduction is the most important factor)

articular surface reconstruction is desirable, but not necessary for a good clinical outcome

How well did you know this?
1
Not at all
2
3
4
5
Perfectly
109
Q

What is the mangement of SCIWORA?

A

MRI to rule out hematoma/edema/cord transection or missed ligamentous injury

Immoblization (c-collar) 2-3 weeks

monitor

How well did you know this?
1
Not at all
2
3
4
5
Perfectly
110
Q

Extrinsic tendon insertions on the thumb

A
  • EPL control IP flexion
How well did you know this?
1
Not at all
2
3
4
5
Perfectly
111
Q

what is true about the prognosis of SCIWORA?

A

latent presentation can occur

delayed is often more severe

worse at presentation=worse prognosis

MRI - can indicate full transection, hemorhage, edema or nothing (the less injury means better prognosis)

How well did you know this?
1
Not at all
2
3
4
5
Perfectly
112
Q

What is a pelligrini-steida lesion

A

chronic MCL deficiency leads to calcification next to medial femoral epicondyle

How well did you know this?
1
Not at all
2
3
4
5
Perfectly
113
Q

What is the stulberg classification?

A

LCP

I - spherical congruency

II - spherical congruency with <2mm loss and starting to have changes

III - aspherical congruency with > 2mm and changes

IV - aspherical congruency with flat head and abnormal neck; mild OA

V - aspherical incongruency; abormal head with severe OA

How well did you know this?
1
Not at all
2
3
4
5
Perfectly
114
Q

What are the treatment of LCP based on stages?

A

A - Observe, casting

B, B/C - containment in fragmentation phase

Later can do abduction/ext or chiari as salvage procedures

How well did you know this?
1
Not at all
2
3
4
5
Perfectly
115
Q

complications of patellar ORIF

A
  • hardware irritation
  • hardware migration is rare
  • stiffness
    • strength
    • pain
    • not associated with immobilization
  • Nonunion
  • deep infection
  • association with OA is unclear
How well did you know this?
1
Not at all
2
3
4
5
Perfectly
116
Q

What are the indications for surgical fixation of a burst fracture?

A

Burst - anterior and middle column invovlement with retropulsion into the canal

<!--StartFragment-->

Neurological deficits
Unstable pattern

Posterior injury
Progressive kyphosis

30 kyphosis
>50% height
>50% canal comprimise

****There is no association of retropulsion with neurological comprimise; the retropulesed fragment if not causing neuro deficiet can be left as the fragment will eventually resorb

<!--EndFragment-->

How well did you know this?
1
Not at all
2
3
4
5
Perfectly
117
Q

What cells produce hyalouronic acid?

A

human synovial cells - special cells in the lining of the synovium

How well did you know this?
1
Not at all
2
3
4
5
Perfectly
118
Q

What are blocks to reductions for subtalar dislocations?

A

Medial - peroneals, EDB, TN capsule

Lateral - FHL, FDL, Tib post

How well did you know this?
1
Not at all
2
3
4
5
Perfectly
119
Q

What is the Hawethorne affect?

A

Patients change their outcome based on the treatment they receive

How well did you know this?
1
Not at all
2
3
4
5
Perfectly
120
Q

When can you stop seeing a patient?

A

When they have a new physician

How well did you know this?
1
Not at all
2
3
4
5
Perfectly
121
Q

What’s the most important factor for TKA ROM

A

pre-op ROM

How well did you know this?
1
Not at all
2
3
4
5
Perfectly
122
Q

Describe Odriscoll’s test for PLRI

A

forearm full supination; supination + extension

flexion + valgus force + axial load = rotatory supination torque on forearm

How well did you know this?
1
Not at all
2
3
4
5
Perfectly
123
Q

Characteristics of endcondroma

A
  • A benign chondrogenic lesion caused by an abnormality of chondroblast function in the physis
    • chondroblasts escape the physis and proliferate in metaphysis
  • 2nd most common benign cartilage lesion (osteochondroma is most common)
  • Age & location
    • age range is 10-60 years
    • occurs in medullary cavity in diaphysis or metaphysis
    • hand is most common location (60%)
      • presentation as pathological fractures common
    • other locations include the femur (20%) and proximal humerus (10%)
  • solitary enchondroma
    • 1% chance of malignant transformation (chondrosarcoma)
  • Ollier’s disease (multipe enchondromatosis)
    • no genetic predisposition has been identified
    • multiple enchondromas in long bone diaphysis
    • 10-30% chance of malignant transformation into chondrosarcoma
    • however associated with high rate of both skeletal and non-skeletal malignancies
    • involved bones are dysplastic
  • Maffucci’s syndrome
    • no genetic predisposition has been identified
    • multiple enchondromas and soft-tissue angiomas
    • 100% chance of malignant transformation into chondrosarcoma
    • increased risk of visceral malignancies (astrocytoma, GI malignancy)
How well did you know this?
1
Not at all
2
3
4
5
Perfectly
124
Q

Stages of fracture healing (MCQ: What cells are present in the inflammatory healing)

A
  • Inflammation
    • Hematoma forms and provides source of hemopoieitic cells capable of secreting growth factors.
    • Osteocytes undergo apoptosis, macrophages, neutrophils and platelets release several cytokines
      • this includes PDGF, TNF-Alpha, TGF-Beta, IL-1,6, 10,12
      • lack of TNF-Alpha (ie. HIV) results in delay of both enchondral/intramembranous ossification
    • Fibroblasts and mesenchymal cells migrate to fracture site and granulation tissue forms around fracture ends
    • during fracture healing granulation tissue tolerates the greatest strain before failure
    • Osteoblasts and fibroblasts proliferate
    • inhibition of COX-2 (ie NSAIDs) causes repression of runx-2/osterix, which are critical for differentiation of osteoblastic cells
  • Repair
    • Primary callus forms within two weeks
      • the mechanical enviroment drives differentiation of either osteoblastic (stable enviroment
        • <2% strain = osteoblasts = intramembranous
          2-10% strain = chondrocytes = enchondral
      • cartilage production provides provisional stabilization
      • Type II collagen (cartilage) is produced early in fracture healing and then followed by type I collagen (bone) expression
      • Amount of callus is inversely proportional to extent of immobilization
  • Remodeling
    • Begins in middle of repair phase and continues long after clinical union
    • chondrocytes undergo terminal differentiation
    • complex interplay of signaling pathways including, indian hedgehog (Ihh), parathyroid hormone related peptide (PTHrP), FGF and BMP
    • type X collagen types is expressed by hypertrophic chondrocytes as the extraarticular matrix undergoes calcification
    • proteases degrade the extracellular matrix
    • cartilaginous calcification takes place at the junction between the maturing chondrocytes and newly forming bone
    • subsequently, chondrocytes become apoptotic and VEGF production leads to new vessel invasion
    • newly formed bone (woven bone) is remodeling via organized osteoblastic/osteoclastic activity
    • Shaped through
      • Wolff’s law: bone remodels in response to mechanical stress
      • piezoelectic charges : bone remodels is response to electric charges: compression side is electronegative and stimulates osteoblast formation, tension side is electropostive and simulates osteoclasts
How well did you know this?
1
Not at all
2
3
4
5
Perfectly
125
Q

What are indications for hemi-epiphyseodesis in congential scoliosis?

A

<5 yo

no kyphosis

<5 segments

hemivertebrae (NOT unsegmented bars)

absence of neurological deficiet

How well did you know this?
1
Not at all
2
3
4
5
Perfectly
126
Q

What is the herring classification?

A

LCP - lateral pillar

A - no involvement

B - > 50% lateral pillar height

B/C - lateral pillar at 50%

C - < 50% lateral pillar height

How well did you know this?
1
Not at all
2
3
4
5
Perfectly
127
Q

What occurs in a normally loaded joint

A
  • NO EDGE LOADING
  • Functional loads will not result in pain during normal ROM
  • Small change in direction of functional loads will not result in sudden shift in the joint
How well did you know this?
1
Not at all
2
3
4
5
Perfectly
128
Q

What are the normal and acceptable criteria for a distal radius fracture?

A

<!--StartFragment-->

  • AP
    • Radial height
      • 13 mm
      • _<5 mm shortening _
    • Radial inclination
      • 23 degrees
      • _change <5° _
    • Articular stepoff
      • congruous
      • _<2 mm stepoff _
  • LAT
    • Volar tIlt
      • 11 degrees
      • dorsal angulation <5° or within 20° of contralateral distal radius

<!--EndFragment-->

How well did you know this?
1
Not at all
2
3
4
5
Perfectly
129
Q

What views are required to assess hillsachs

A

IR view

stryker notch

How well did you know this?
1
Not at all
2
3
4
5
Perfectly
130
Q

How do you prevent heel pad migration in a symes amputation?

A

attach heel pad to anterior tibia

How well did you know this?
1
Not at all
2
3
4
5
Perfectly
131
Q

What is true about HIT?

A
  • Immune repsonse to heparin 4-10 days following initiation
    • less response to LMWH, but present
    • type 1 is not immune mediated
  • skin lesions, chills, fever, dyspnea, chest pain
  • HIT as likley diagnosis
    • timing (4-10 days following start)
    • >50% drop in platelets
    • presence of DVT with heparin
  • start WU if >50% drop in platlets
    • functional assay
    • immunoassay
    • U/S to rule out DVT
    • CT PE
  • treatment
    • stop heparin/LMWH
    • don’t use warfarin until platelets normal
    • Dabigatron or direct platelet inhibiter a good options
    • avoid platelet transfusion
How well did you know this?
1
Not at all
2
3
4
5
Perfectly
132
Q

what are the radiographic findings of a VISI?

A

scaphoid flexion

triquetrum extesion

volar flexion lunate

No LT widening

SL angle <30

break in gilula’s arc with superior migration of triquetrum

How well did you know this?
1
Not at all
2
3
4
5
Perfectly
133
Q

What would PRC be an option for?

A

Stage 2 SNAC

Stage 2 SLAC

Stage IIIb, IV Keinbochs

failed implant

severe trauma to lunate

How well did you know this?
1
Not at all
2
3
4
5
Perfectly
134
Q

Options for surgery in congenital scoliosis

A
  • in situ arthrodesis, anterior/posterior or posterior alone
    • indications
      • unlateral unsegmented bars with minimal deformity
    • As risk for crackshaft and may not prevent progression in children
  • hemiepiphysiodesis
    • indications
      • intact growth plates on the concave side of the deformity
      • Cannot use in a patient with unsegmented bars
      • <5 yrs. with curve <40-50
    • mixed results
  • osteotomy
    • osteotomy of bar
    • for more severe; rigid deformities
  • hemivertebrectomy
    • hemivertebrae with progressive curve causing truncal imbalance and oblique takeoff
      • often caused by a lumbosacral hemivertebrae
    • <6 yo
    • flexible curve > 40 degrees
    • Difficulty procedure requiring both anterior and posterior exposure
  • spinal column shortening resection
    • indications
      • deformities that present late and have severe decompensation
      • rigid, severe deformities
      • pelvic obliquity, fixed
How well did you know this?
1
Not at all
2
3
4
5
Perfectly
135
Q

What are the portals for elbow arthroscopy?

A
  • Proximal anterolateral
    • 2cm proximal, 1cm anterior to lateral epicondyle
    • Radial n.
  • Distal anterolateral
    • 1 cm anterior and 1-3cm distal to lateral epicondyle
    • 1st portal for supine position
    • See radial head, medial side of elbow, coronoid, trochlea, brachialis insertion, coronoid fossa
    • Radial and lateral antebrachial cutaneous
  • Direct lateral (or midlateral)
    • “soft spot” portal (in triangle formed by olecranon, radial head, epicondyle)
    • Initial site for joint distension before scope is inserted, viewing posterior compartment (capitellum, radial head, radioulnar articulation)
    • relatively safe, lateral antebrachial cutaneous nerve
  • Anteromedial
    • 2 cm anterior and 2cm distal to medial epicondyle.
    • Place under direct visualization.
    • medial antebrachial cutaneous and median
  • Proximal medial (or superomedial)
    • 2cm proximal to medial epicondyle, anterior to intermuscular septum
    • viewing entire anterior compartment, radial head, capitellum, coronoid, trochlea
    • ulnar and median
  • Straight posterior (transtriceps)
    • 3cm proximal to olecranon, triceps midline (musculotend. junction)
    • Elbow partially extended, good for removing impinging olecranon osteophytes and loose bodies from posteromedial compartment
    • posterior antebrachial cutaneous and ulnar nerve
  • Posterolateral
    • 2-3 cm proximal to olecranon and just lateral to triceps
    • center of anconeus triangle
    • Elbow 20-30deg flexion (to relax triceps)
    • Best access to posterior compartment, radiocapitellar joint (debridement of OCD capitellum), olecranon fossa and posterior structures
    • posterior antebrachial cutaneous and medial brachial cutaneous and ulnar
How well did you know this?
1
Not at all
2
3
4
5
Perfectly
136
Q

What are the conditions associated with this pathology?

A

thumb hypoplasia/radial clubhand

VACTERL
Holt-Oram
thrombocytopenia-absent radius (TAR)
Fanconi anemia

How well did you know this?
1
Not at all
2
3
4
5
Perfectly
137
Q

What are the transfers for an MC nerve palsy?

A

Regain elbow flexion

Stindler’s flexorplasty

latissiumus dorsi/pec major to biceps

How well did you know this?
1
Not at all
2
3
4
5
Perfectly
138
Q

What are the findings of hypophosphatasia?

A
  • Low ALP
  • High Ca
  • Normal PTH
How well did you know this?
1
Not at all
2
3
4
5
Perfectly
139
Q

What are the contents of the carotid sheath?

A

the common carotid artery as well as the internal carotid artery
internal jugular vein
the vagus nerve (CN X)
the deep cervical lymph nodes

How well did you know this?
1
Not at all
2
3
4
5
Perfectly
140
Q

What is characteristic of scoliosis associated with neurofibromatosis?

A
  • Idiopathic vs dystrophic
    • idiopathic is more common
    • overall, idiopathic will become dystrophic in 65%
    • younger age=higher risk
    • dystropic progress faster
  • usually thoracic kyphoscoliosis (type II curve)
    • type I curve < 50 deg kyphosis
  • short, segmented, sharp curve with distorted ribs
  • XR
    • short, segemented
    • sharp curve
    • vertebral scalloping
    • rib pencilling (suseptible to dislocation)
    • enlarged foramina
  • MRI
    • dural ectasia
    • dumbell lesion
    • paraspinal mass
    • vertebral scalloping
  • TX = bracing ineffective
    • early in young children (cobb <20)
    • >20 = fusion
How well did you know this?
1
Not at all
2
3
4
5
Perfectly
141
Q

What is the strongest construct for a patella fracture

A

figure of 8 wires with cannulated screws

interfrag screws for comminution

How well did you know this?
1
Not at all
2
3
4
5
Perfectly
142
Q

What are the boundaries of hunter’s canal and what are located with-in?

A

vastus medius, adductor magnus, sartorius

femoral artery, femoral vien, saphenous nerve, nerve to vastus medialis

How well did you know this?
1
Not at all
2
3
4
5
Perfectly
143
Q

Etiology of scapulothoracic crepitus

A
  • 43% - osteochondroma
  • 30% - idiopathic
  • 15% - rib resection
  • 12% - soft tissue
How well did you know this?
1
Not at all
2
3
4
5
Perfectly
144
Q

What is the definition of incidence?

A

of people patients who developed the disease in a set period of time

How well did you know this?
1
Not at all
2
3
4
5
Perfectly
145
Q

What is the most important alignment to maintain in subtalar fusion?

A

5 deg valgus

How well did you know this?
1
Not at all
2
3
4
5
Perfectly
146
Q

Regarding human bite, how do you treat Eikenella Corodins?

A

Penecillin

How well did you know this?
1
Not at all
2
3
4
5
Perfectly
147
Q

What are acceptable treatments of this lesion?

A
  • >25% deficit
    • Infraspinatous sewn into defect (remplissage)
  • >40%
    • Allograft for young patient
    • hemi for older patient
How well did you know this?
1
Not at all
2
3
4
5
Perfectly
148
Q

Characteristics of transient osteoporosis

A
  • young, middle age men
    • or women in 3rd trimenster
  • Presents with rapid development of pain and reduced ROM
  • Differential
    • regional migratory OP
    • AVN
    • neoplasia
  • usually in the hip; ONLY lower extremity
    • thought to be similar to an AVN process that stops
  • blood tests will be normal but should be done to rule out other causes
  • XR
    • will show demineralization over 3-6 weeks
    • 2 years to return, but never collapses
  • MRI
    • will rule out AVN
How well did you know this?
1
Not at all
2
3
4
5
Perfectly
149
Q

Compare medial and lateral OCD lesions of the talus

A
  • medial talar dome
    • usually no history of trauma
    • more common
    • more posterior
    • larger and deeper than lateral lesions
  • lateral talar dome
    • usually have a traumatic history
    • more superficial and smaller
    • lower incidence of spontaneous healing
    • more anterior
How well did you know this?
1
Not at all
2
3
4
5
Perfectly
150
Q

What nerve guides the dissection into the popliteal fossa?

A

medial sural cutaneous nerve

How well did you know this?
1
Not at all
2
3
4
5
Perfectly
151
Q

What is true about THA versus cannulated screws?

A

No increase mortalilty at 3 years

Better function outcome with THA

Increased risk of complications with THA

Reduced risk of re-operation with THA

How well did you know this?
1
Not at all
2
3
4
5
Perfectly
152
Q

What are the sources of compression of the PIN?

A

Fibrous Bands

Recurrent radial artery - lesh of henry

Tendonous ECRB

Arcade of Frosch (proximal supinator) (MOST)

Distal supinator (least)

How well did you know this?
1
Not at all
2
3
4
5
Perfectly
153
Q

What is the pathology of SIADH and what is the treatment

A
  • ​dilution of plasma from water retention and therefore low NA
  • hyponatremia = headache, N/V, confusion
    • but do not become volume deplete
  • treated with water restriction
  • Must be differentiated from cerebral salt wasting
    • loose Na in the urine due to renal insufficiency and become hyponatriemic and hypovolemic
    • treat with aggressive fluid resucitaiton
How well did you know this?
1
Not at all
2
3
4
5
Perfectly
154
Q

Indications for surgical treament of MCL

A

commonly ruptured off the femoral insertion

  • Failed Non-op
  • Excessive > 10o laxity in Full extension (Associated posteromedial injury)
  • Associated Cruciate or Meniscal Injuries
  • Entrapment of MCL ligament inside compartment
  • Stener lesion - can lay over top of pes tendon if comes off the tibia = much less likely to heal
How well did you know this?
1
Not at all
2
3
4
5
Perfectly
155
Q

Approach to radial head arthroplasty

A
  • Approach for Surgery
    • ​kocher will allow you to see LUCL
    • EDC split will all you to see more anterior
    • Arm held in pronation to protect the PIN, 2.6cm distal to the RC joint
  • indications
    • for comminuted fracture with 3 or more fragments
    • technique include two type of metal prosthesis, both are in use
      • monopolar prosthesis are better at resisting residual instability
      • pressfit that acts as a stiff spacer
        • risk of loosening
      • bipolar prosthesis that is cemented into the neck of the radius
        • risk of dislocation
      • silicon replacements are no longer used
  • Sizing radial head
    • Size the contralateral head pre-op
    • Size to original radial head - thickness and cup
    • Trial reduction with visual inspection - most reliable
      • Proximally should line up with less sigmoid notch
      • Lateral UH joint are opposed
        • gapping 0.9mm = 2mm overstuffing
    • Flouro
      • Medial and lateral UH joint lines (can’t see this until >6mm)
      • Look for congruency of the medial UH joint
  • Repair the LCL using the docking technique
    • this is one of the most important parts of the technique
  • complications
    • overstuffing of joint that leads to capitellar wear problems and malalignment instability
How well did you know this?
1
Not at all
2
3
4
5
Perfectly
156
Q

Treatment

A
  • Nonoperative
    • double ring splint
    • indications
      • can prevent hyperextension of PIP
  • Volar plate advancement and PIP balancing with central slip tenotomy
    • indications
      • progressive deformity
    • technique
      • address volar plate laxity with volar plate advancement
      • correct PIP joint muscles imbalances with either
        • FDS tenodesis indicated with FDS rupture
        • spiral oblique retinacular ligament reconstruction
        • central slip tenotomy (Fowler)
  • DIP Arthrodesis
    • Dorsal approach
    • Extensor tenodesis with capsulotomy
How well did you know this?
1
Not at all
2
3
4
5
Perfectly
157
Q

What is the litchman classification?

A
How well did you know this?
1
Not at all
2
3
4
5
Perfectly
158
Q

How do you prevent and treat recurrent laryngeal nerve injury?

A
  • 2-3%, 9% with re-operation
    • can get ENT for second procedure
    • use the same side
  • likley cause is impingement between ET tube cuff and retractors and continuous compression against the esohpagus
  • Presents with post-op hoarsness or vocal cord paralysis
    • don’t forget to include esphophageal rupture in your diagnosis
  • if you see it, watch over time
  • if not improved over 6 weeks than ENT consult to scope patient and inject teflon
How well did you know this?
1
Not at all
2
3
4
5
Perfectly
159
Q

What are risk factors for clavicle nonunion?

A

smoking
advancing age
female
comminution
> 100% displacement

> 2cm shortened
location = lateral 1/3 fractures - medial to CC ligaments are most unstable and have highest risk of nonunion

How well did you know this?
1
Not at all
2
3
4
5
Perfectly
160
Q

What is the number needed to treat

A

the number to prevent one bad outcome

How well did you know this?
1
Not at all
2
3
4
5
Perfectly
161
Q

What is hastings classification?

A

Dorsal PIP dislocation

  • Type I-Stable
    • <30%; stable
  • Type II-Tenuous
    • 30-50%-if reducible in flexion, dorsally based extension block splint
  • Type III-Unstable
    • >50%-ORIF, hamat autograft, or volar plate arthroplasty
  • Becomes unstable after 40% because they loose the attachments of the collaterals
How well did you know this?
1
Not at all
2
3
4
5
Perfectly
162
Q

Compare TEA and ORIF for distal humerus fractures

A
  • No difference in complication rate
  • Negligable data to suggest TEA decreases reoperation
  • Main difference is that TEA has improved functional outcomes (Level 1 evidence)
How well did you know this?
1
Not at all
2
3
4
5
Perfectly
163
Q

What are the common compliations of halo

A
  • Loosening (36%)
    • pins placed more cranial = more loosening
      • just above pinna at equator of head
      • 0.5cm above brow
    • ring should be closer to head
    • pins should engage at 90 deg angle
    • metal rings better than graphite ring
  • Infection (20%)
    • especially with anterior pin in temporalis fossa because
    • treat infection with oral antibiotics if pin not loose
    • if infection with loose pin then remove pin
  • Discomfort (18%)
    • treat by loosening skin around pin
  • Dural puncture (1%)
  • Crainal nerve VI palsy
How well did you know this?
1
Not at all
2
3
4
5
Perfectly
164
Q

Appropriate measures to deal with scare resources

A
  • Choosing interventions known to be beneficial on the basis of evidence of effectiveness.
  • Minimizing the use of marginally beneficial tests, such as the diagnostic zebra-hunt.
  • Minimizing the use of marginally beneficial interventions, such as the latest generations of antimicrobials for common infections.
  • Seeking the least costly tests or treatments that will accomplish the diagnostic or therapeutic goal.
  • Using the natural queue, treating patients in order of appearance unless morally relevant considerations of need and benefit require modification of this approach. - apparently this is flawed
  • Ranking patients with whom you have an established patient-doctor relationship ahead of unknown or future patients.
  • Supporting rather than opposing reasonable efforts to conserve health care resources.
  • Avoiding manipulation of the rules of the health care system to give unfair advantage to your own particular patients.
  • Resolving conflicting claims for scarce resources justly, on the basis of morally relevant criteria of need and benefit.
  • Employing fair and publicly defensible procedures for resolution of conflicting or competing claims.
  • Seeking resolution of unacceptable shortages at the level of hospital management (meso allocation) or through political action at the level of government (macro allocation).
  • Informing your patients of the impact of cost constraints on care in a humanistic way, as a matter of respect for persons. Embittered blaming of administrative or governmental systems during discussions with the patient at the point of treatment should be avoided.
  • Developing guidelines for individualization in the face of uncertainty in order to promote a reasonable balance between individual choice and systemic cost control.
How well did you know this?
1
Not at all
2
3
4
5
Perfectly
165
Q

What is the prognosis of a neonatal brachial plexus injury?

A

90% resolve by 2 years

Recovery of biceps by 2 months is good prognosis

How well did you know this?
1
Not at all
2
3
4
5
Perfectly
166
Q

Clinical findings associated with muchopolysaccharoidosis

A
  • typically none at birth. After a period of normal growth
  • coarse facial features
  • fullness of skin/brows
  • enlarged tongue
  • retained epicanthal folds
  • short neck
  • hydrocephalus
  • corneal clouding
  • inguinal/umbilical hernias
  • hepatosplenomegaly
  • otitis media, conductive hearing loss
  • chronic resp tract infections
  • cardiac d/o, carpal tunnel syn.
How well did you know this?
1
Not at all
2
3
4
5
Perfectly
167
Q

What nerve is most at risk with the FCR approach?

A

palmar cutaneous branch of the median nerve

How well did you know this?
1
Not at all
2
3
4
5
Perfectly
168
Q

What it the treatment for traumatic axillary nerve plasy

A

monitor, most will resolve on their own

EMG at 3 months to assess recovery with operative exploration if no evidence of recovery

NOTE - stabs wounds should be explored eariler, and post-surgical can consider EMG eariler

How well did you know this?
1
Not at all
2
3
4
5
Perfectly
169
Q

Diagnosis?

A

multiple hereditary exostosis (MHE)

  • mutations affect the prehypertrophic chondrocytes of the physis
  • genetics
    • autosomal dominant
    • mutations in EXT1, EXT2, and EXT3 genes (tumor suppressor genes)
  • EXT1 mutation have a more severe presentation compared to patients with the EXT2 mutation including
    • higher rate of chondrosarcoma
    • more exostoses
    • more limb malalignment with less forearm and knee range of motion
    • more pelvic and flatbone involvement
  • prognosis
    • 5%-10% malignant transformation to chondrosarcoma in patients with MHE
    • proximal lesions more likely to undergo malignant transformation than distal lesions
    • most common location of secondary chondrosarcoma is the pelvis (usually occur as low-grade chondrosarcomas)
How well did you know this?
1
Not at all
2
3
4
5
Perfectly
170
Q

When are you most likely to injury superior gluteal nerve?

A

Glut med split with hardinge

How well did you know this?
1
Not at all
2
3
4
5
Perfectly
171
Q

Common tendon transfers for RA?

A
  • EIP to EPL
  • EIP to EDC5
  • EDC4/5
    • EIP to EDC5
    • 3/4 side-side
  • EDC3/4/5
    • 2/3 side-side
    • EIP - 4/5 side-side
  • EDC2/3/4/5 without EIP
    • use FDS
How well did you know this?
1
Not at all
2
3
4
5
Perfectly
172
Q

What factor is important to outcome of a PIP fracture-dislocation

A
  • reduced joint is the most important thing
  • anatomic reduction is less important
How well did you know this?
1
Not at all
2
3
4
5
Perfectly
173
Q

What is CREST syndrome

A
  • Member of the Scleroderma family
  • Calcinosis
  • Raynaud phenomenon
  • Esophageal dysmotility
  • Sclerodactyly
  • Telangiectasia
How well did you know this?
1
Not at all
2
3
4
5
Perfectly
174
Q

What are the pros and cons of using plaster?

A
  • Pros
    • ​decrease pressure sore
    • better ability to conform to limb
  • ​Cons
    • ​low strength to weight ratio
    • poor resistance to water
How well did you know this?
1
Not at all
2
3
4
5
Perfectly
175
Q

What is the best predictor of OA in an ankle fracture?

A

posterior malleolus fracture

How well did you know this?
1
Not at all
2
3
4
5
Perfectly
176
Q

What is true about BMP2 in open tibia fractures?

A

lower need for future bone graft

lower rate subsequent soft tissue

lower infection rate (only in Gustilo 3 pattern)

How well did you know this?
1
Not at all
2
3
4
5
Perfectly
177
Q

Indications for exploration of the radial nerve

A
  • Open fracture
  • High-velocity gunshot or penetrating injury
  • Vascular injury
  • Nerve deficit after closed reductiona Distal third (Holstein-Lewis) fractures
How well did you know this?
1
Not at all
2
3
4
5
Perfectly
178
Q

Pathology of scapular winging

A
  • Serratus anterior - medial
    • long thoracic (C5, 6, 7)
  • Trapezius - lateral
    • accessory
  • Rhomboids - lateral
    • dorsal scapular
  • Treatment
    • trapezium strengthening
    • eden lange transfer - lateralize levator scapulae and rhomboids
    • scapulothoracic fusion
How well did you know this?
1
Not at all
2
3
4
5
Perfectly
179
Q

Indications for delayed closure with VAC or bead pouch

A
  • limited soft tissue viability
  • lack of soft tissue coverage
  • severe contamination
  • Note - bead pouch or abx PMMA can significantly decreased risk of infection with delayed closure
How well did you know this?
1
Not at all
2
3
4
5
Perfectly
180
Q

What are the advantages and disadvantages of a single incision technique for distal biceps?

A

asthetic

less HO

10% improved strength with double

more often LCFN traction injury with single

How well did you know this?
1
Not at all
2
3
4
5
Perfectly
181
Q

Prevention of HIV transmission

A

always double glove

waterproof barriers

instrument tie or staples

pass sharp instruments in a bowl

JAAOS 1996

How well did you know this?
1
Not at all
2
3
4
5
Perfectly
182
Q

What are the blocks to a lateral subtalar dislocation?

A

<!--StartFragment-->

posterior tibialis tendon
flexor hallucis longus
flexor digitorum longus

<!--EndFragment-->

How well did you know this?
1
Not at all
2
3
4
5
Perfectly
183
Q

Acceptable criteria for distal radius malunion?

A
How well did you know this?
1
Not at all
2
3
4
5
Perfectly
184
Q

Describe the retroperitoneal approach

A
  • Considerations
    • transperitoneal has a higher risk of retrograde ejaculation (superior hypogastric plexus)
    • if you go from left it’s easier, because easier to mobilize arteries rather than frail vessels
    • Common Iliac branch at L4
  • Incise the rectus and transversalis
  • Find the arcuate line (fasica of the rectus); incise it and use it to retract the peritoneum
  • Iliolumbar vessles branch off the internal iliac and need to be ligated for exposure to L4/5
  • For exposure to L5/S1 the median sacral artery needs to be identified and ligated
How well did you know this?
1
Not at all
2
3
4
5
Perfectly
185
Q

What is the classficiation of thumb hypoplasia?

A

I - minimal

II - hypoplasia of musculature

IIIA - tendon abnormal, hypoplastic MC, stable CMC

IIIB - tendon abonormal, hypoplastic MC, unstable CMC

IV - floating thumb

V - absent thumb

How well did you know this?
1
Not at all
2
3
4
5
Perfectly
186
Q

What is the classification for lesser arc injuries?

A

Purely ligamentous - Mayfeild

Stage 1 - SL ligament

Stage 2 - space of poirer (lunocapitate lig)

Stage 3 - UTCC/UL

Stage 4 - lunate dislocation (dorsal radiolunate ligament)

How well did you know this?
1
Not at all
2
3
4
5
Perfectly
187
Q

What is the treatment of an adult galeazzi fracture?

A
  • Usually operative, requires operative fixation as it requires anatomic fixation of the DRUJ
    • volar approach to the radius
      • restoration of the radial bow is key
    • dorsal approach to the DRUJ
      • make sure to examine TFCC if does not reduce with the radius
      • typically avulsed off the ulna
  • Treatment of the ulna
    • immobilization in supination (6 weeks)
      • indicated if DRUJ stable following ORIF of radius
    • percutaneous pin fixation
      • indicated if DRUJ reducible but unstable following ORIF of radius
      • cross-pin ulna to radius in supination
      • examine the TFCC
      • leave pins in place for 4-6 weeks
    • open surgical reduction
      • indicated if reduction is blocked
      • suspect interposition of ECU Tendon
      • Examine and repair TFCC if still unstable
    • open reduction internal fixation
      • indicated if a large ulnar styloid fragment exists
      • fix styloid and immobilize in supination
How well did you know this?
1
Not at all
2
3
4
5
Perfectly
188
Q

What are the radial nerve tendon transfers?

A

Lat dorsi/deltoid/biceps to triceps

PT to ECRB (Or ECRL to ECRB side to side if PIN)

FDS/FCR/FCU to EDC

PL/FDS to EPL

189
Q

important findings on emg

A
  • used to measure the intrinsic electrical activity of muscles
    • activity at rest
    • voluntary contraction
    • following stimulation
  • can characterize myopathy versus denervation
    • complex repetative DC = myopathy
    • fasiculations and fibrillations = denervation
  • Early following denervation will not see fibrillations
    • can assess chronicity
  • Affected by needle position, age, temp
190
Q

What are complications of the Darrach procedure?

A

Used primarily in older patients:

Ulnar drift

painful ECU subluxation

radio-ulnar impingement

instability of the ulnar shaft

191
Q

What is the treatment of MFH?

A
  • wide resection
  • chemo

(treated like a bone tumor)

192
Q

What lesions are associated with internal impingement of the shoulder

A
  • Lesion
    • fraying of posterior rotator cuff
    • posterior and superior labral lesions
    • hypertrophy and scarring of posterior capsule glenoid (Bennet lesion)
    • cartilage damage at posterior glenoid
  • Associated with GIRD
193
Q

What are the clinical features of larsen syndrome?

A
  • Genetics - autosomal dominant (AD) and recessive (AR) inheritance patterns
    • AD linked to a mutation of the gene encoding filamin B
    • AR linked to carbohydrate sulfotransferase 3deficiency
  • Normal intellegence
  • Multiple dislocations (hyperlaxity)
  • clubfoot
  • cervical kyphosis (60%)
    • clinical important because often requires fusion for instability
    • can present as myelopathy or with UE weakness
  • scoliosis
  • spatulate thumbs
194
Q

Acceptable alignment for humeral shaft fracture and contraindications to coaptation splinting

A

  • contraindications to functional bracing
    • severe soft tissue injury or bone loss
    • unreliable patient
    • polytrauma
    • brachial plexus injury
    • proximal one-third humeral fracture
    • inability to maintain reduction (segmental fracture)
    • radial nerve palsy is NOT a contraindication to functional bracing
195
Q

What are the characteristic head findings of LCP?

A

Coxa magna

Coxa plana

short femoral neck

196
Q

What are causes of a swan neck finger?

A

***imbalance of the musculature

FDS tear - overpull of FDP

mallet finger - ext at PIP

intrinsic contracture

MCP volar subluxation

197
Q

What deformity are you at risk for if you use an extramedullary tibial cutting guide

A

varus

198
Q

Goals of reduction for a pediatric femur fracture

A

< 10° of coronal plane
< 20° of sagittal plane deformity
no more than 2cm of shortening
< 10° of rotational malalignment

199
Q

Pathology of this hand and tests

A
  • Intrinsic minus hand (or tight extrinsics)
  • causes
    • ​ulnar nerve palsy
      • cubital tunnel syndrome
      • ulnar tunnel syndrome
    • median nerve palsy
      • Volkmann’s ischemic contracture
      • leprosy (Hansen’s disease)
    • failure to splint the hand in an intrinsic-plus posture following a crush injury
    • Charcot-Marie-Tooth disease (hereditary motor-sensory neuropathy)
    • compartment syndrome of the hand
  • ​Pathoanatomy
    • ​​loss of intrinsics
      • leads to loss of baseline MCP flexion and loss of IP extension
    • strong extrinsic EDC
      • leads to unopposed extension of the MCP joint
      • remember the EDC is not a significant extensor of the PIP joint
      • most of the MCP extension forces on the terminal insertion of the central slip come from the interosseous muscles
    • strong FDP and FDS
      • leads to unopposed flexion of the PIP and DIP
  • ​​Tests
    • ​ulnar nerve worse in 4/5
    • median nerve worse in 2/3
    • ​if the MCP is flexed, the IP joints will extend
      • opposite of bunnels test
200
Q

What are the zones of nec fasc and what is the required treatment?

A
  • Touniquette; aggressive debridement of all affected tissues including questionable tissues; meticulous hemostasis
  • Zone 1
    • diagnosis (histology); grey, dead fascia with dishwater pus, foul smelling, hemorhhagic bullae with necrotic skin; send culture from here
  • Zone 2
    • diagnosis (cultures); woody erythema with small bullae (potentially salvagable); can estabilish border with blunt finger disection
  • Zone 3
    • healthy tissue, need to resect to this level with a margin of 5-10mm
  • Post-excision
    • no VAC; silver dry dressing; inspect wound with-in 24 hours
201
Q

What is the most important prognostic factors of soft tissue sarcoma?

A

1) mets (distant mets the most important factor for any staging system)
2) tumour grade (represents a tumours ability to metastasize)
3) size
4) compartmentalization
5) tumour depth- controversial as a prognosticator

202
Q

What is your cutoff for bunions?

A

IMA <15, HVA <30 = distal

IMA > 15, HVA > 30 = proximal

hypermobile first ray = lapidus

arthritis or progressive = fusion

congruent with high DMMA (>20) = consider double osteotomy

203
Q

Treatment of a child with flexible hindfoot and cavus foot

A

plantar release

Tendon transfer

dorsiflexion first ray

+/- jones

204
Q

What is true about the RA spine?

A

Affect synovial joints (there are 22)

C1-C2 can lead to both basilar invag and atlantoaxial instability

Lateral masses can be affected and contribute to AA instability

205
Q

What is true about bone healing in OI?

A

Same healing time, weaker bone

206
Q

What is are the generations of cement?

A

First - hand packed, no plug

Second - cement plug, gun, canal preparation with brush and adrenaline

Third - vacum, retrograde filling, pressurization, centralizer

207
Q

How is the lunate interossoeous pressure affected by wrist motion

A

pressure goes up with both flexion and extension

208
Q

What is frederichs disease?

A

aseptic osteonecrosis of the medial clavicle

red joint

ESR/CRP/WBC/Rheum work-up are normal

MRI - necrotic bone islands

209
Q

Classification of Hallux Rigidus

A

Cougling and Shurnas

  • Changes the arch of use due to dorsal osteophytes, >10 deg of motion is not needed for a normal gait, and in fact push-off improves following fusion in these patients
  • Pathology due to increased strain across the MTP joint
  • Grade 0
    • Stiffness with normal XR
  • Grade 1
    • mild pain at extremes of motion
    • mild dorsal osteophyte, normal joint space
  • Grade 2
    • moderate pain with range of motion increasingly more constant
    • moderate dorsal osteophyte,
  • Grade 3
    • significant stiffness, pain at extreme ROM, no pain at mid-range
    • severe dorsal osteophyte, >50% joint space narrowing
  • Grade 4
    • significant stiffness, pain at extreme ROM, pain at mid-range of motion
    • same as grade III
210
Q

What are the deforming forces on this fracture?

A

Bennet fracture

Abductor pollicis longus

extensor pollicis longus

adductor pollicis

211
Q

What are the indications to surgically stablize a 1st MCP UCL tear?

A
  • >10-15 deg varus/valgus
    • compared to contralateral side
  • grade 3 tear (will open in extesion and flexion)
    • >35 deg of opening
      • extension - accessory
      • 30 deg flexion - proper
    • no end point
  • stener lesion
    • may be able to palpate a lesion

US and MRI can help aid the diagnosis, but usually the diagnosis is made on physical exam

212
Q

What is the defining feature of juvinile hallux valgus?

A

Increased DMMA

213
Q

What are the common tumours of the posterior elements?

A

ABC

osteoblastoma

osteoid osteoma

ostosarcoma

214
Q

What are conditions associated with syndactyly?

A

Acrosyndactyly (ring syndrome)

Poland syndrome

Apert syndrome

Carpenter syndrome (acrocephalopolysyndactyly)

215
Q

What are 4 tests for UMN lesions above the waist

A
  • Inverted Brachioradialis
    • tap on BR
    • flexion of thumb and index finger
  • Hoffmann’s reflex
    • forced flexion of MF DIPJ
    • reflex finger and thumb flexion is elicited
  • Scapulohumeral Reflex
    • tapping tip of spine of scapula
    • brisk elevation of scapula & abduction of the humerus
    • positive in > 95% of myelopaths
  • Jaw Jerk
    • a cerebellar sign
    • distinguishes myelopathy from cerebellum
216
Q

What are the features of unilateral facet dislocation?

A

flexion/distraction injury, with rotation

4/5 or 5/6

25% anterior translation (50% is bilateral)

SP don’t line up on AP

bow-tie sign

217
Q

What is true about wound closure in a Gustilo 3a wound closure?

A

Increased wound complication rates with delayed closure (not necessarily infection)

218
Q

What are the risk factors of SCFE bilateral slip?

A

Endocrinopathy (hypothyroid)

renal osteodystrophy

growth hormone

219
Q

Characteristics and treatment of DeQuervains

A
  • Common overuse syndrome
  • Less common than trigger (2-8/1000)
  • Can be a significant cause of loss of work in labourers
  • Best treatment is rest, NSAIDS and splinting
  • May require release of the first compartment
220
Q

What are indications for MRI in patients with AIS?

A

L thoracic curve

rapid progression (>1deg/month)

acute angle

neuro sx (abdominal reflex - tethered cord, sringomyelia, tumor, dysraphysm)

absence of rotation (syrinx = tumour, myelomeningocele)

pain

221
Q

What cells synthesize hyaluronic acid?

A

synoviocytes

222
Q

What structures are present in each of the acetabular quadrants for screw placement - which is safe and which is dangerous?

A
223
Q

Prevelance of nonunion in humeral shaft fracture treatment

A
  • defined as lack of union after 6 months
    • 2 to 10% in nonoperative managment
    • 15% with primary ORIF
  • risk factors
    • axial distraction on injury films represents high level of soft tissue injury and increase chance of nonunion
    • open fx
    • unstable or segmental fx
    • infection
    • initial treatment with hanging cast
    • shoulder or elbow stiffness (motion directed to fracture site)
    • patient factors (smoking, obesity, alcoholism, malnutrition, noncompliance)
  • treatment
    • compression plating with bone grafting
      • shown to be superior to both IM nailing with bone grafting and compression plating alone
      • use anterolateral approach (allows exploration of radial nerve)
    • vascularized fibula bone graft and compression plating
      • indicated if > 6 cm bone defect
      • technique
        • use bone graft as an intramedullary dowel (1-2 cm inside each end)
        • stabilize with 4.5 dynamic compression plate
      • peroneal artery (fibula) is anastomosed to brachial artery
      • peroaneal vein anastomosed with basilic or cephalic vein
224
Q

Clinical and radiographic features that suggest C-spine instability?

A
  • Clinical
    • neck pain
    • midline tenderness
    • pain with rotation/flexion
    • neurological findings
  • Subtle changes on XR
    • soft-tissue swelling
    • hypolordosis
    • disk-space narrowing or widening
    • widening of the interspinous distances
    • Angulation >11 deg between vertebrae
    • Saggital translation >3.5mm
225
Q

What are the intervals for the Smith-Pete?

A

TFL-sartorius

Rectus - glut med

226
Q

What are common causes complications with CTR?

A
  • Pillar Pain (25%) most common cause of pain post-op)
    • pain between triquetrum and hamate
  • Palmar cutaneous branch
  • Recurrent CTR due to inadequate release (7-20%)
    • most common complication of endoscopic procedure
    • Inadequate proximal transverse ligament release with scarring of the nerve
    • inadquate distal release
  • Other
    • recurrent motor branch
    • hypertrophic scar
    • laceration of superficial arch
227
Q

What is the ASIA spinal score?

A

A - complete

B - sensation but not motor

C - < grade 3 > 50% levels

D - > grade 3 > 50% levels

E - sensory only

228
Q

What are type I and type II error

A
  • Type 1
    • null hypothesis is rejected even though it is true
    • by definition, alpha-error rate is set to .05, meaning there is a 1/20 chanc a type-I error has occured
    • Bonferroni correction
      • post-hoc statistical correction made to P values when several dependent or independent statistical tests are being performed simultaneously on a single data set
  • Type 2
    • a false negative difference that can occur by detecting no difference when there is a difference or accepting a null hypothesis when it is false and should be rejected
    • power = 1 - (type-II error)
    • Clinical significance
      • a study that fails to find a difference may be because
        • there actually is no difference or
        • the study is not adequately powered
229
Q

What allows for increase in strength during resistance training?

A

recruitment of more motor units

230
Q

Other indications for radial head arthroplasty?

A
  • Indications
    • comminuted radial head fracture
    • nonunion/maluion
    • instabilty (essex-lopraseti)
    • rheumatoid or osteoarthritis
    • tumour
  • ​Contraindications
    • repariable fracture
    • capitellar arthrosis
231
Q

Outcomes of total disc replacement?

A

2006 study showed that replacement is as good as ALIF with 50% success rate. Single level fusion for isolated back pain is controversial at best, and disc replacement has been shown to have inferior results when provocative discography is positive…some think that single level with disease free adjacent levels is the indication

232
Q

What is the classification for monteggia fractures?

A
  • BADO
  • I - anterior
    • Pronation injury
    • Reduced with flexion and supination
  • II - posterior
    • Supnation injury
    • Reduced with extension and pronation
  • III - lateral
    • Direct lateral injury
    • Reduced with flexion and supination
  • IV - # rad/ulna
233
Q

What are the indications for ORIF of MC #?

A
  • 2.5mm shortening
  • malrotation
  • comminution
  • angulation 10/20/30/40
  • multiple MC fractured (leads to instability)
234
Q

What is the presentation of a hamate fracture and what are the best images?

A

Usually golfers or raquet with ulnar wrist pain

+/- nerve sx

carpal tunnel view

CT IS BEST TO MAKE DIAGNOSIS

235
Q

What structure is most important following an Lateral UCL tear?

A

coronoid

236
Q

What nerve is commonly injured in posterior iliac crest grafting?

A

cluneal

237
Q

Presentation and treatment of congential trigger thumb

A
  • get an increase FPL thickening (normal sheath) with triggering at A1
    • 25% are bilateral
    • presents with a fixed deformity
    • > 2yo = resolution unlikley
    • Usually not painful
    • Notta’s nodule = flexor tendon prominence
  • passive extension exercises and observation
    • indications
      • usually considered first line of treatment
      • not recommended for fixed deformities in older children
    • technique
      • passive thumb extension exercises
      • duration based on clinical response
    • outcomes
      • 30-60% will resolve spontaneously before the age of 2 years old
        • extension splinting
    • indications
      • consider alongside stretching regime
      • not recommended with fixed deformities in older children
    • technique
      • splints maintain IP joint hyperextension and prevent MCP joint hyperextension
      • duration for 6-12 weeks
    • outcomes
      • 50-60% resolution in all age groups
      • high drop out rate from therapy
  • A1 pulley release
    • indications
      • fixed deformity beyond age of 12 months of age
      • failed conservative treatment
    • outcomes
      • 65-95% resolution in all age groups
238
Q

Characteristics and treatment of trigger finger

A
  • Characteristics
    • 2-3% of the general population
    • ring finger most commonly affected
    • usually idipathic but more common in
      • dibetics
      • RA
      • amyloidosis
  • night splinting, activity modification, NSAIDS
    • first line of treatment
  • steroid injections
    • best initial treatment for fingers, not for thumb
    • technique
      • give 1 to 3 injections in flexor tendon sheath
      • diabetics do not respond as well as non-diabetics
  • surgical debridement and release of the A-1 pulley
    • in cases that fail nonoperative treatment
  • release of A1 pulley and 1 slip of FDS (usually ulnar slip)
    • pediatric trigger finger
    • presents with Notta’s nodule (proximal to A1 pulley), flexion contracture and triggering
    • may need to release remaining FDS slip and A2 or A3 pulley as well
239
Q

What is true regarding decompression of the ulnar nerve

A
  • simple decompression (in situ, neurolysis) is the best
  • If you need to transpose, subcuteanous is better
    • failed in situ
    • throwing athlete
    • poor nerve bed - tumor, HO
    • After stiff elbow
      • <90 bend
      • >30 deg motion release
240
Q

What is the most common level of ped slip?

A

L5-S1 (L5 nerve root is most affected)

241
Q

What is the treatment of thumb hypoplasia?

A

I - no treatment

II/IIIA - tendon correction, oppenoneplasty with UCL recon

IIIB - ablation with pollicization OR 2nd toe/toe joint transfer

IV - ablation with pollicization OR 2nd toe transfer

V - pollicization or 2nd toe transfer

242
Q

What fibers are most important to prevent hoop stress?

A

circumferntial

243
Q

What are the foot deformities by myelomeningocele level?

A
244
Q

What is the strongest fixation for a lateral mall fracture?

A

posterior anti-glide plate with lag screw

especially superior in weak bone

245
Q

What is true about Dupytrens’

A

<!--StartFragment-->

  • A benign proliferative disorder characterized by fascial nodules and contractures of the hand
    • starts as nodules in the hand, pregresses to fibrous bands and then contractures of MCP, moving distally
  • Epidemiology & genetics
    • autosomal dominant with variable penetrance
    • 5-7th decade of life with 2:1 male to female ratio
    • highest incidence in Caucasian males of northern European descent
  • Pathophysiology
    • myofibroblast is the dominant cell type
    • cytokines have been implicated
    • ectopic manifestations
      • Ledderhose disease (plantar fascia)
      • Peyronie’s disease (dartos fascia of penis)
      • **Garrod disease (knuckle pads) **
  • Associated conditions
    • HIV, alcoholism, diabetes, antiseizure medications
  • Prognosis
    • long-term recurrence rate after operative management up to 50%

<!--EndFragment-->

246
Q

Compare T-test, Chi Square, ANOVA and Linear Regression

A
  • Continuous data
    • _​_potentially infinite possible values (ie. Height, HEAD SIZE)
  • Categorical data
    • _​_variables are a given number of categories (ie. Gender, fracture classification)
  • T-test
    • used to compare TWO independent CONTINUOUS variables
  • Chi square
    • used to compare two or more CATEGORICAL variables
  • ANOVA
    • used to compare three or more CONTINUOUS Variables
  • Linear Regression
    • used to investigate whether a variable can be predicted by another single variable (simple linear regression) or by a combination of other variables (multiple linear regression)
247
Q

Relative indication for operative fixation of humeral shaft fractures

A
  • Open fracture
  • Associated articular fracture Neurovascular injury
  • Floating elbow
  • Impending pathologic fracture Polytrauma
  • Failure of closed management
248
Q

Indications for observation of congenital scoliosis

A

<!--StartFragment-->

Block vertebrae
incarcerated hemivertebrae
nonsegmental hemivertebrae
some partially segmented hemivertebrae

<!--EndFragment-->

249
Q

What is true about more than one screw in SCFE?

A

high risk of penetration

higher risk of AVN

Better torsional control

250
Q

What casues swan neck deformity in RA?

A

MCP joint subluxation

Mallet finger

FDS tear

tight intrinsics

251
Q

What is the most common cause of shoulder instability following TSA?

A

insufficient subscap repair

252
Q

What is the main reason for failure of a tension band construct for olecranon fracture?

A

wire passed superficial to triceps

Nonuion, failure and necessary hardware removal are common complications

Technique - 1.2mm wire, 3-4 cm distal to # drill 2.0mm hole in olecranon, k-wires parallel into anterior cortex, wire must pass under triceps on periosteum proximally (use an angiocath)

253
Q

Characteristics and treatment of MC fractures

A
  • deformity
    • 5mm rotation = 1.5cm rotation at finger
    • 2mm short = 7 deg ext lag
  • neck fractures
    • multiple MC
    • 10/20/40/60
    • comminution
    • open fracture
  • transverse/short oblique
    • if short oblique is non-displaced can treat with cast immobilization
    • apex dorsal due to long flexors and interosseous
    • easy to reduce with flexion to of MCP 90 deg; difficult to maintain
    • transverse usually need to be reduced, if you can reduce it then ok to cast, otherwise need to fix
    • easy to control with “Ex-fix” - pinning to other MC
      • DC at 3-4 weeks
  • long spiral
    • difficult to maintain closed due to shortening and rotation
      • if not displaced; can treat in cast
      • if it’s displaced, you can’t reduce it without ORIF
    • fix wtih lag screws
      • one perpendicular to fracture, one perpendicular to bone
      • if fracture < 2X cortical diameter then neutralize with plate
    • wear intrinsic plus splint during high risk situations; but can start moving right away
    • ORIF
      • 2-5mm shortening (5mm in JAAOS 2000)
      • malrotation
      • comminution
      • multiple
      • 10/20/30/40
254
Q

When would you brace a congential scoli

A

when they have a flexible compensatory curve

255
Q

What are the levels of evidence

A
  • Level 1
    • Randomized controlled trial (RCT)
    • Meta-analysis of randomized trials with homogeneous results
  • Level 2
    • Prospective comparative study (therapeutic)
    • Meta-analysis of Level 2 studies or Level 1 studies with inconsistent results
  • Level 3
    • Retrospective cohort study
    • Case-control study
    • Meta-analysis of Level 3 studies
  • Level 4
    • Case series
  • Level 5
    • Case report (a report of a single case)
    • Expert opinion
    • Personal observation
256
Q

What is the use and structure of the SF-36

A
  • a generic, multi-purpose, short-form health survey consisting of 36 questions
  • useful for
    • surveys of general and specific populations
    • comparing the relative burden of diseases
    • differentiating the health benefits produced by a wide range of different treatment
  • Self-rated preinjury pain-related disability is a predictor of moderate to severe pain 6 months after musculoskeletal injury as measured by SF-12
  • consists of 8 scaled scores, which are the weighted sums of the questions in their section
  • transformed into a 0-100 scale on the assumption that each question carries equal weight
    • vitality
    • physical functioning
    • bodily pain
    • general health perceptions
    • physical role functioning
    • emotional role functioning
    • social role functioning
    • mental health
257
Q

What predicts OA post-ACL?

A

chondral lesion (strongest)

obesity

medial meniscectomy

258
Q

Key points of metatarsus primus varus

A
  • Adduction deformity of the great toe characterized by
    • deformity at metatarsophalangeal joint
    • short thick 1st metatarsal
    • firm band-like abductor hallucis muscle
  • Associated conditions
    • often associated with polydactyly – duplication of the great toe
  • Must be differentiated from metatarsus adductus
    • Lateral border of the foot is normal
    • More rigid than MA
  • Early casting is used, associated with hallux valgus
  • Severe deformity is indication for osteotomy
259
Q

How can you avoid malreduction when nailing a distal tibial fracture

A
  • improved outcomes with reaming
    • ​may even be better than plates
  • no difference with weight bearing early
  • avoid eccentric reaming
  • stabilize posterior malleolus first
  • blocking screws
  • bone reduction forceps
  • distal locking screw first with appropriate technique
  • femoral distractor
  • unicortical plates
  • multiple screws
  • multiple planes
  • use of a VAC can decrese edema
260
Q

What are 3 syndromes associated with langerhans histiocytosis

A
  • **Eosinophilic granuloma (EG) **
    • usually a single self-limited lesion found in younger patients
  • **Hand-Schuller-Christian disease (HSC) **
    • chronic, disseminated form with bone and visceral lesions
    • also known as Langerhans cell histiocytosis with visceral involvement
  • **Letterer-Siwe disease (LSD) **
    • fatal form that occurs in young children
261
Q

What are your anatomical landmarks for positioning your humoral hemiarthroplasty

A
  • Normal alignment
    • 140 deg Neck shaft angle
    • 30 deg retroversion
  • humeral head is 5.6cm above pec major
    • >10mm proud = increased tuberosity failure
    • 15mm shortening tolerated
  • Use bicipetal groove or long head biceps to access version compared to fin
    • 20-30 deg retroversion
  • Tuberosity placement 1-1.5cm distal to superior margin
    • wire or sutures can augment fixation
262
Q

What are common cause of bacterial infection

A
  • GAS - gram + in chains
    • think nec fasc
  • Clostridium - gram + rods
    • think nec fasc
  • Staph - gram + clusters
263
Q

What are important radiographic measurments for blounts

A

<!--StartFragment-->

  • findings suggestive of Blounts disease
    • varus focused at proximal tibia
    • severe deformity
    • asymmetric bowing
    • progressing deformity
    • sharp angular deformity
    • lateral thrust during gait
    • metaphyseal beaking
    • different than physiologic bowing which shows a symmetric flaring of the tibia and femur
  • measurements
    • metaphyseal-diaphyseal angle (Drennan)
      • >16 ° = blounts
      • 10-16 is controversial and should be observed
      • <10 ° = normal
    • tibiofemoral angle
      • angle between the longitudinal axis of the femur and tibia

<!--EndFragment-->

264
Q

What are the bands associated with dupytrens

A
  • pretendinous band
  • spiral band
  • natatory band/ligament
    • causes contracture of the web space lateral and reduces abduction
  • lateral digital sheet
  • Grayson’s ligament

pulls the NV strucutre central and superficial​

265
Q

When would you consider radial head excision? What are the complications?

A
  • indications
    • older patients with limited demands
    • in a delayed setting for continued pain of an isolated radial head fracture
  • contraindications
    • forearm ligament injury (identify with radius pull test)
    • coronoid fracture
    • MCL deficiency
  • complications
    • muscle weakness
    • wrist pain
      • proximal migration of radius leads to ulanr positive variance and ulnocarpal abutment
    • valgus elbow instability
    • heterotopic ossification
    • arthritis
266
Q

Treatment of recurrent posterior shoulder dislocation

A
  • 50% are caused by trauma
  • More common to have recurrent subuluxation that wears the posteior gleniod
  • May present with recurrence of pain more than instability, but can be quite limited
  • Some are traumatic, some are habitual
    • habitual will not do well with surgery
  • Physical
    • sulcus, jerk, posterior stress, load and shift
  • Most will benefit from strengthening dynamic stabilizers
  • Can benefit from surgery when nonsurgical fails (ideally recurrent, post-traumatic, unilateral)
    • Posterior Bankart lesion
      • Arthroscopic or open posterior Bankart repair
    • Excessive capsulolabral laxity
      • Arthroscopic or open posterior capsular shift ± rotator interval closure
    • Glenoid erosion
      • ​Posterior glenoid bone grafting
    • Increased glenoid retroversion >20
      • Posterior opening wedge glenoid osteotomy
  • Complications
    • recurrence
    • subscapular nerve injury
      • if you split the IS more than 1.5cm medially
    • stiffness from overtightening
    • may be associated with future OA
267
Q

What is a mannerfelt lesion?

A

Rupture of FPL in RA with-in the carpal tunnel due to scaphoid ostophyte, volar synovitis

Treated with FDS transfer or pullthrough, resect spicule, or PIP fusion

Important to rule out AIN palsy

268
Q

What is the most common complication of elbow dislocation?

A

lack of terminal extension

269
Q

What complication can occur post-op AIS with posterior fusion and oliguria?

A

SIADH

270
Q

What are the types of nerve injury?

A
271
Q

What is the treatment of pathological fracture of enchondroma?

A

Allow to heal

then curettage and bone graft

increase complications associated with acute treatment

272
Q

What is the primary stabilizer to the PIP joint?

A

collateral ligaments

The volar plate resists joint hyperextension, while the collateral ligaments are the primary restraints to motion in the coronal plane
At terminal extension, the volar plate and the accessory collateral ligaments assume a larger role.
In flexion, the proper collateral ligament is tightened over the flare of the condyle and becomes the primary stabilizer against lateral displacement.
Both the volar plate and at least one collateral ligament must be injured for dislocation of the PIP joint to occur

273
Q

What is true about bilateral TKA?

A

increased PE, cardiac complications, need for transfusions and mortality

lower infection, lower DVT

274
Q

Classification and treatment for this fracture?

A

HASTINGS

  • Type I-Stable
    • <30%-treat with dorsally based extension block splint
  • Type II-Tenuous
    • 30-50%-if reducible in flexion, dorsally based extension block splint
  • Type III-Unstable
    • >50%-ORIF if possible, hamat autograft/dynamic ex-fix, or volar plate arthroplasty
  • Comminuted type C
    • ​Dynamic distraction external fixation
  • ​Chronic
    • ​fusion
275
Q

What are the risk factors for distal radius instability

A
  • age of 80
  • metaphyseal comminution
  • 5mm shortening
  • dorsal angulation IS NOT
    • according to Mcqueen study
276
Q

What is are the properties of PMMA

A
  • becomes strongest at 24 hours
  • provides adhesion to bone (not to the prosthesis)
  • weak in shear and tension, strongest in compression
  • decreasing porosity increases strength by 15%
277
Q

What is the appropriate timing and options for brachial plexus injury in an adult?

A
  • Sharp Injury
    • repair immediately
  • Blunt injury
    • tag the nerve ends acutely and repair or graft at 3-4 weeks (this is after they have demarcated)
  • If concern of root avulsion
    • 3-6 weeks (nerve graft or transfer)
    • graft targeted for specific function (sural)
      • C5 - shoulder abduction (axially or suprasca)
      • C6 - elblow flexion (MC)
      • C7 - elbow and wrist extension (rad)
    • neurotization (transfer)
      • spinal accessory
      • intercostal
      • medial pectoral
      • phrenic
      • ulnar (oberlin)
  • Neuropraxia - 3-4 months if no re-innervation (>12 months has poor outcomes)
    • PT and splinting to maintain ROM of the joints
    • EMG at 3-4 weeks (when wallarian degeneration will occur)
    • then repeat to see if improving or follow clinically
      • decreased fibrillations and increased AP (but may not correlate so still need to check clinically)
  • >12 months - secondary procedures
    • tendon transfers
    • osteotomy
    • arthrodesis
278
Q

What percent of cortical blood flow is disrupted with osteotomy?

A

50%

279
Q

What is fredrichs disease?

A

AVN of the medial clavicle

atraumatic dislocation

often occurs in females

medial bowing of the clavicle

280
Q

What is the composition of an intervertebral disc?

A
  • Annulus Type 1 collagen
  • Nucleus is Type 2 collagen
  • 30% proteoglycans
  • Aging disc
    • decrease in
      • nutritional transport
      • water content
      • absolute number of viable cells
      • proteoglycans
      • pH
    • increase in
      • an increase keratin sulfate to chondroitin sulfate ratio
      • lactate
      • degradative enzyme activity
    • no change in
      • absolute quantity of collagen
281
Q

What is the most sensitive neurlogical exam for carpal tunnel?

A

sensory changes - shows prolonged sensory latency

282
Q

Differential for medial elbow pain?

A
  • UCL rupture
  • ulnar neuritis
  • golfers elbow
  • medial epicondyle epophysitis
  • valgus extension overload
  • OCD capitellum
  • olecranon stress fracture
  • ****elbow OA is NOT a cause of medial elbow OA
283
Q

What is the approach to an isolated cervical epidural abcess?

A

posterior decompression

284
Q

What is true about UBC?

A

Chance of fracture increases with size of cyst

(<10%) rarely heal with fracture

steroid injection equivalent to bone aspirate

no malignant transformation

curettage with bone graft when failed or with lower extremity

285
Q

Options to restore arthrogrypotic extension contracture

A
  • triceps to biceps
  • steindler flexorplasty
  • lat dorsi to biceps
  • posterior release with triceps lengthening pec major to biceps
286
Q

Prognosis of progression of congenital scolisosis: worst to best

A
  • unilateral bar, contralateral hemivertebrae (5-10deg)
  • double hemivertebrae
    • although depends on whether on opposite sides or not
  • unilateral bar (5-6 deg)
  • segemented hemivertebrae (2-5deg)
  • unsegmented hemivertebra
  • incarcerated hemivertebra
  • unincarcerated hemivertebra
  • block vertebrae
287
Q

Indiations for pec major tear non-op repair

A

partial tear

proximal tears (sternoclavicular origin)

Older/sedentary individuals

288
Q

What are the sites of compression of the median nerve?

A

Ligament of struthers

biceps aponeurosis (lacertus fibrosis) - AIN

heads of pronator - AIN

FDS arcade - AIN

accessory FPL - AIN

thrombosed artery - AIN

289
Q

What would the bone quality be like in an iliac crest graft from a child with OI?

A
  • thin cortex
  • decreased cancellous bone
  • increased fibrous tissue
  • Radiographs:
    • thin cortex
    • osteopenia
    • sabre shins
    • wormian bones
  • Histology:
    • increased diameter haversion canals,
    • increased osteoblasts, osteoclasts
    • replicated cement lines
    • less trabeculae
    • thin cortices
290
Q

What is the classification for syndactyly?

A

Simple - soft tissue only

Complex - fusion of adjacent phalanges

Complicated - abnormal bone

Complete vs incomplete - complete extends to fingertips

291
Q

Anatomy of the Spinal Column

A
  • Descending Tracts (motor)
    • lateral corticospinal tract (LCT)
      • UMN are in the lateral portion of the white matter
      • They synapse with anterior horn cells (ventral) in the grey matter, and more central portion of the spinal cord
    • ventral corticospinal tract
      • Rubriospinal
        • smaller, less axons
        • voluntary muscle control
        • primarily flexion (decorticate)
        • extra-pyramidal
      • vestibulospinal
        • extension
        • extrapyramidal
        • postural
  • Ascending tracts (sensory)
    • Synapse with the doral root ganglion and enter via the posterior horn of the grey matter (doral)
    • dorsal columns
      • deep touch
      • vibration
      • proprioception
    • lateral spinothalamic tract (LST)
      • pain
      • temperature
    • ventral spinothalamic tract (VST)
      • light touch
292
Q

What is the treatment for swan neck deformity

A
  • Double ring splint
    • can try this first always
  • volar plate advancement and PIP balancing with central slip tenotomy
    • indications
      • progressive deformity
    • technique
      • address volar plate laxity with volar plate advancement
      • correct PIP joint muscles imbalances with either
        • FDS tenodesis indicated with FDS rupture
        • spiral oblique retinacular ligament reconstruction
        • central slip tenotomy (Fowler)
  • DIP Arthrodesis
    • Dorsal approach
    • Extensor tenodesis with capsulotomy
293
Q

What is your EMG threshold for cubital tunnel?

A

<!--StartFragment-->

conduction velocity <50 m/sec across elbow

<!--EndFragment-->

294
Q

What is the classification for clavicle fractures?

A

Group 1 - middle, most common

Group 2 - distal (Neer)

1 - lateral (non-op)

2a - medial (sux)

2b - medial with conoid (sx)

3 - intra-articular

4- physeal

5 - comminuted

Group 3 - proximal (anterior, posterior)

295
Q

What is the best study group to capture prevalence?

A

Cross sectional

296
Q

What ligaments are torn in a VISI deformity?

A

lunotriquetral

dorsal radiocarpal

long and short radiolunate

297
Q

What is the treatment of a delayed SH2 colles?

A

Delays (10 days), even severe, do not re-reduce. Mould the cast and hope to remodel. Already starting to heal so can do osteotomy later if you need to

298
Q

What intra-opeartive complications are you worried about in duschennes muscular dystrophy

A
  • malignant hyperthermia is common intraoperatively
    • pretreat with dantrolene
  • intraoperative cardiac events
299
Q

What are the tendon transfers for an ulnar nerve palsy?

A

FDS/ECRB to Adductor

APL/ECRL/EIP to 1st interosseous

FDS/ECRL to lateral band of ulnar digits (must pass volar to the transferverse intermetatarsal ligament)

300
Q

What are the pros and cons of ceramic bearings

A
  • Pros
    • best wear characteristics
    • particles are non-oncogenics
  • Cons
    • risk of fracture
      • 1/5000-1/12000
      • 70% with-in 12 months
      • higher with small, neutral offset heads
    • Chipping 1.2%
    • Squeak 0.45%
    • Stripe wear with edge loading
    • Fewer prosthetic options
    • More technically demanding
301
Q

What are complications associated with distal biceps repair

A
  • LACN most commonly
  • PIN and superficial radial nerve
    • two incision technique was supposed to reduce this, but now with smaller anterior incision it is low with both
  • synosteosis
  • HO
302
Q

What is the last physis to fuse?

A

medial clavicle

303
Q

What position do you get a painful, audible snap in ECU tendonitis

A

supintation (and extension)

  • get a tear in the sheath of the 6th compartment
  • supnation it subluxes out of the sheath and reduces with pronation
  • US can assess dynamic stability, MRI can show tendonisits
  • Usually treated with a splint, if fails can repair the sheath
304
Q

What is the pathophysiology of muchopolysacharoidosis and treatment

A

GAGs accumulate in lysosomes of chondrocytes, synovium → disorganized physeal architecture → 2° inflammatory response → irregularly shaped metaphyses → skel dysplasia

  • Treatment
    • IV enzyme replacement
    • bone marrow transplant
  • Improved cardiac/resp, improved hearing and resolution of hydrocephalus
  • no effect on orthopedic manifestations
305
Q

What are the theories of charcot foot?

A
  • Insensate foot with repetative microtrauma
  • Autonaumic dysfuction causes hyperemia, resorption and osteopenia
  • Inflammatory cytokines and TNF-alpha
306
Q

Windswept hand deformity characteristics?

A

**Occurs in distal arthrogryposis

  • Flexion, adduction of thumb (thumb in palm)
  • narrowing first web space
  • flexion/ulnar deviation of fingers
307
Q

What is the veiw of choice for traumatic SC dislocation

A

obtain serendipity views ( beam at 40 cephalic tilt)

anterior dislocation - affected clavicle above contralateral clavicle
posterior dislocation - affected clavicle below contralateral clavicle

308
Q

Describe the anatomy of the distal biceps

A
  • Biceps inserts on radial tuberosity
    • 21 mm total
    • long head more for supnation
      • Proximal and radial
    • Short head more for flexion
      • Distal and ulnar
      • Lacertus originates from the short head
  • Complete vs Partial
    • Parital usually occurs on the radial side
  • Intersubstance transection
  • Mechanism of tear
    • eccentric contraction at 90 deg flexion
309
Q

What are the 5 ligaments of the interosseous membrane of the forearm

A

central band is key portion of IOM to be reconstructed
accessory band
distal oblique bundle
proximal oblique cord
dorsal oblique accessory cord

310
Q

What is the jupiter modification of monteggia fractures

A
  • A - intra-articular coronoid
  • B - extra-articular coronoid
  • C - extra-articular shaft
  • D - intra-articular comminuted
311
Q

What are the forms of muchopolysaccharidoses?

A
  • Morquio syndrome
    • Accumulation of keratin sulfate
    • Type A - (galactosamine-6-sulfate-sulphatase deficiency)
    • Type B - (beta-galactosidase deficiency)
  • Hurler syndrome - worst
    • dermatin sulfate
    • alpha-L iduronidase deficiency
    • bone marrow transplant in first year of life reduces mortality but not orthopedic manifestations
  • Sanfilippo syndrome - most common
    • heparan sulphate
  • Hunter syndrome
    • dermatin/heparin sulfate
    • sulpho-iduronate-sulphatase deficiency
312
Q

What is an appropriate technique for valgus intertroch osteotomy

A
  • indicated in patients after femoral neck nonunion
    • verticle fracture (Pawel C)
    • can be done even in presence of AVN, as long as not severely collapsed, but may want to consider MRI preop
  • Plan your osteotomy for the fracture to be 30 deg from horizontal
    • choose appropriate blade plate
  • First K-wire for blade goes into the head at your planned site
  • second K-wire at the osteotomy should be 2 cm distal to this
  • Try not to medialize the canal
    • decreases offset, reduces abductor strength
    • increase joint reaction force
313
Q

Etiology and Pathoanatomy

A

Swan Neck Deformity

  • Characterized by
    • hyperextension of PIP
    • flexion of DIP
  • Caused by
    • lax volar plate
    • imbalance of muscles forces on PIP (extension force > flexion force)
  • Injuries include
    • MCP joint volar subluxation (rheumatoid arthritis)
    • mallet finger
    • FDS laceration
    • intrinsic contracture
  • Primary lesion is lax volar plate that allows hyperextension of PIP
    • trauma
    • RA
    • Hyperlaxity
  • Secondary lesion is imbalance of forces on the PIP joint (PIP extension forces that is greater than the PIP flexion force)
    • mallet injury
      • leads to transfer of DIP extension force into PIP extension forces
    • FDS rupture
      • leads to unopposed PIP extension with an intact FDP
    • intrinsic contracture
      • tethering of the lateral (collateral) bands by the transverse retinacular ligament as a result of PIP hyperextension.
      • if the lateral (collateral) bands are tethered, excursion is restricted and the extension force is not transmitted to the terminal tendon, and is instead transmitted to the PIP joint
    • MCP joint volar subluxation
      • caused by rhuematoid arthritis
314
Q

What is the consequence of plating a fibula short?

A

Lateral ankle instability (can occur with or without syndesmosis injury)

315
Q

What is true about ACL femoral tunnel placement?

A

** proper placement **

    1-2 mm rim of bone between tunnel and posterior cortex of femur

** anterior misplacement**

    leads to a knee that is tight in flexion and loose in extension

    occurs from failure to clear "residents ridge"

**posterior misplacement** (over-the-top)

    leads to a knee that is lax in flexion and tight in extension
316
Q

Algorythm for fracture fixation through burns

A
  • If fracture can be treated no-op …. splint ensuring wound access possible
  • If fracture needs OR
    • Definitive treatment ideal
    • Through alternate skin incision if possible
    • If incision must be through burn, must do this < 48hrs from injury
    • If >48hrs (tissue will be colonized):
      • Ex-fix (until wounds healed)
      • Splint (until wounds healed)
      • ORIF at the same time of wound debridement and skin grafting
317
Q

What is the most appropriate treatment for a comminuted inferior pole patellar fracture

A
  • debridement with tendon re-attachement to the extra-articular surface
  • don’t take more than 40% of the patella
  • +/- circlage wire re-inforcement
318
Q

What structure is disrupted with extensor tendon dislocation at the MCP?

A

Saggital band

319
Q

What is the best bone graft to use in fibrous dysplasia?

A

Cortical allograft (cortical is slower to incorperate)

320
Q

What are options for foot orthoses?

A
  • Flares
    • can help with partial foot amputation
    • fixed deformity
    • ankle instability
  • Shank - rigid lever that comes into toes
    • amputation
    • hallux rigidis
  • Mild rocker
    • mild metatarsalgia
  • In most of the questions around a lisfranc amputation the only appropropritate awnswer is an AFO
  • Heel-Toe Rocker
    • hammer toe
    • ankle fusion
    • midfoot amputation
  • Toe -only
    • proprioception
  • Severe angle
    • ulcerations
  • Double rocker
    • charcot foot
321
Q

What is the order of priority when treating brachial plexus injuries?

A
  1. Elbow flexion
  2. shoulder abduction/stability
  3. hand sensibility
  4. wrist extension
  5. finger flexion
  6. wrist flexion
  7. finger extension
  8. intrinsic hand function
322
Q

What is the algorythim for treament of open tibia fractures

A
  • Abx, tetanus
  • Abx as soon as possible
  • IM Nailing
    • has been shown to be safe initially for treatment gustilo I-IIIb
    • no consensus reamed vs unreamed
  • Ex-fix
    • If ex-fix required change to IM nail as soon as possible
  • Plates not recommended
  • Indications for Ex-fix
    • gross conatmination
    • delay to treatment
323
Q

What is the consequence of resussitating a pateint with NS

A

hypercholermic metabolic acidoisis

324
Q

Describe powers ratio

A

Basion to Posterior C1 / Opisthion to Anterior C1

(CD/BA)

325
Q

What is true about SCIWORA (Spinal cord injury without radiographic abnormality)?

A

Children < 8yo

Column can stretch without injury (>2 inches), but causes damage to the cord (<1cm)

most common in c-spine

50% are delayed presentation

(some thought that better emerg care and more immobilization has decreased delayed injury to cord)

326
Q

Unstable PIP fractures can be treated conserativley. How can you define this?

A
  • Dorsal
    • 30-50%,
    • >50% OR >30 flexion to maintain = unstable
  • _​_Volar
    • >50% OR unable to maintain with extension
  • Pilon
    • either stable or grossly unstable
327
Q

What are the non-orthopedic manifestations of OI

A

blue sclera
hearing loss
brownish opalescent teeth (dentinogenesis imperfecta)
wormian skull bones (puzzle piece intrasutural skull bones)
Thin skin, prone to bruising
Cardiac manifestations
increased risk of malignant hyperthermia

328
Q

Compare IM nail to plate fixation for distal tibia fractures

A
  • Plates have lower malunion rate
    • better for a very distal fracture
      • use anterolateral and MIPO
    • cause hardware irritation
  • Nails better for elderly patients with thin skin and poor soft tissues
    • fracture blisters, diabetes, open
    • cause knee pain
  • How to increase nail stability
    • locked screws
    • more screws
    • multi-planar screws
329
Q

What are the releases for a varus knee?

A

** osteophytes**

deep MCL

Semi- M - moslty extension, should be released last

SMCL - anterior or posterior based on whether you need release

PCL - flexion

330
Q

What is the blood supply to the patella?

A

anterior to quads, posterior to the patellar tendon

form a ring around the patellar and penetrate anteriorly

avoid anterior soft tissue stripping

25% necrosis rate with fracture

331
Q

What factor is the most predictive of DRUJ arthrosis or instability following distal radius fracture

A

change in ulnar varience (ie radial height)

332
Q

What is the last bone in the foot to ossify?

A

Navicular

333
Q

Indications for patellar fracture fixation

A
  • >4mm distraction
  • >2mm stepoff
  • intra-articular fragments
  • osteochondral fragment
  • lose of extensor mechanism

no data exist on post-op rehab protocols

334
Q

Best position for ankle fusion

A

5 deg valgus

neutral dorsiflexion

5 deg ER

335
Q

Options for surgery for congenital scoliosis

A

<!--StartFragment-->

  • Spinal fusion<!--StartFragment-->
    • indications
      • In young patients anterior fusion needed to prevent progression or crankshaft
      • young patients with significant progression, neurologic deficits, or declining respiratory function
        • girls < 10 yrs
        • boys < 12 yrs
      • patients with failure of formation with contralateral failure of segmentation at any age
    • technique
      • nutritional status of patient must be optimized prior to surgery<!--EndFragment-->
    • in situ arthrodesis, anterior/posterior or posterior alone
      • unlateral unsegmented bars with minimal deformity
    • hemiepiphysiodesis
      • intact growth plates on the concave side of the deformity
      • patients less than 5 yrs. with < 40-50 degree curve
      • mixed results
    • osteotomy
      • osteotomy of bar
    • hemivertebrectomy
      • hemivertebrae with progressive curve causing truncal imbalance and oblique takeoff
      • often caused by a lumbosacral hemivertebrae
      • patients < 6 yrs. and flexible curve < 40 degrees best candidates
    • spinal column shortening resection
      • deformities that present late and have severe decompensation
      • rigid, severe deformities
      • pelvic obliquity, fixed

<!--StartFragment-->

  • growing rod construct
    • indications
      • may be used in an attempt to control deformity during spinal growth and delay arthrodesis
    • outcomes
      • need to be lengthened approximately every 6 months for best results
  • VEPTR (vertical expanding prosthetic titanium rib)
    • indications
      • thoracic insufficiency syndrome
    • outcomes
      • long-term follow up is needed to determine efficacy

<!--EndFragment-->

<!--EndFragment-->

336
Q

treatment for ulnar neuropathy

A
  • in situ decompression
    • just as good results with fewer complications compared to transposition
  • decompression with transposition
    • subcutaneous is better than submuscular with fewer complications
  • medial epiconylectomy
    • visibly subluxing ulnar nerve
337
Q

Sites of entrapment of this nerve

A

ULNAR CLAW HAND

  • Associated conditions
    • cubitus varus or valgus deformities
    • medial epicondylitis
    • burns
    • elbow contracture release​
  • most common
    • between the two heads of FCU/aponeurosis (most common site)
    • within arcade of Struthers (hiatus in medial intermuscular septum)
    • between Osborne’s ligament and MCL
  • less common
    • medial head of triceps
    • medial intermuscular septum
    • medial epicondyle
    • fascial bands within FCU
    • anconeus epitrochlearis (anomalous muscle from the medial olecranon to the medial epicondyle)
    • aponeurosis of FDS proximal edge
338
Q

What are common marine organisms

A
  • The most common infections are still staph aureus and strep pyogenes
    • but need extra coverage for the strange bugs
  • A hydrophilia - fresh
  • pseudomonas
  • mycobacterium marinum - swimming pool
    • chronic or occult
  • Vibrio - salt water
339
Q

What is the role of ADH in a trauma patient?

A

Anti-diuretic hormone

Promotes water resorption in kidneys

restores intravascular volume

  • low osmolailty = ADH off
    • ​increase urine output
    • increase osmolality
  • ​High osmolaltiy = AHD on
    • ​decrease urine output
    • decrease osmolaity
  • ​SIADH
    • ​dilution of plasma from water retention and therefore low NA
    • hyponatremia = headache, N/V, confusion
    • treated with water restriction
340
Q

Classification of this injury

A

Jersey Finger

  • Refers to an avulsion injury of FDP from insertion at base of distal phalanx
    • a Zone I flexor tendon injury
    • FDP muscle belly in maximal contraction during forceful DIP extension
  • Ring finger involved in 75% of cases
    • during grip ring fingertip is 5 mm more prominent than other digits in ~90% of patients
    • Weaker insertion point
    • Least independent movement
  • _​_Proximal to A5 there is a dual vascualr supply, treatment will depend on whether one or both are disrupted
    • ​Vinculum Longus Profundus (VLP)
    • Vinculum Longus Superficialis (VSP)
  • Leddy Packer Classification
    • Type I
      • FDP tendon retracted to palm
      • Leads to disruption of the vascular supply
      • Prompt surgical treatment within 7 to 10 days
    • Type II
      • FDP retracts to level of PIP joint
      • VLP intact
      • Attempt to repair within several weeks for optimal outcome
    • Type III
      • Large avulsion fracture limits retraction to the level of the DIP joint (A4)
      • All blood supply intact
      • Attempt to repair within several weeks for optimal outcome
    • Type IV
      • Osseous fragment incarcerated in A4 and simultaneous avulsion of the tendon from the fracture fragment (“Double avulsion” with subsequent retraction of the tendon usually into palm)
      • If tendon separated from fracture fragment, first fix fracture via ORIF then reattach tendon as for Type I/II injuries
    • Type V
      • boney avulsion with comminution
341
Q

What is the equation for power (not powers ratio; the power of a study)

A

Definition

an estimate of the probability a study will be able to detect a true effect of the intervention

Equation

power = 1 - (probability of a type-II, or beta error)

342
Q

What are the predictors of wound healing in lower extremity amputations?

A

ABI > 0.5

02 >30mmHg

albumin > 2.5

WBC > 1500

343
Q

Indications for operative fixation of first time dislocators

A
  • Absolute
    • Associated Injury
    • >50% rotator cuff tear
    • Glenoid osseous defect >25%
    • Humeral head articular surface osseous defect >25%
    • Proximal humerus fracture requiring surgery
    • Irreducible dislocation
    • Interposed tissue or nonconcentric reduction
    • Failed trial of rehabilitation
    • Inability to tolerate shoulder restrictions
    • Inability to perform sport-specific drills without instability
  • Relative
    • >2 shoulder dislocations during the season
    • Overhead or throwing athletes Contact sport athletes
    • Injury near the end of the season Age <20 years
344
Q

What are the findings associated with hans-schuller-christian disease

A
  • diabetes incipidous
  • EG
  • exopthalamos
345
Q

What is the treatment for a VISI deformity?

A

acute - CRPP with dorsal ligament repair

Chronic - LT fusion

If due to ulnocarpal abutment - ulnar shortening with arthroscopic debridement of LT and TFCC

346
Q

What is the extracellular composition of articular cartilage

A
  • water
    • makes up 65% to 80% of mass of the cartilage
    • accounts for 80% of the weight near the surface
    • 65% at the deep zone
    • water content
      • decreases with normal aging
      • increases with osteoarthritis
      • increased water content leads to
        • increased permeability
        • decreased strength
        • decreased Young Modulus of elasticity
  • collagen
    • makes up 10 to 20% of total cartilage mass
    • type II collagen accounting for 90% to 95% of the total collagen content.
    • functions to provide cartilagenous framework and tensile strenth
    • small amounts of types V, VI, IX, X, and XI collagen are also present
  • proteoglycans
    • makes up 10 to 15% of cartilage
    • functions to provide compressive strength and attract water (hydrophilic)
    • produced by chondrocytes
    • composed of GAG subunits and bound to protein core
      • chondroitin sulfate
      • keratin sulfate
  • noncollagenous protein
347
Q

When do you see Hawkin’s sign following a talus fracture?

A
  • 6-8 weeks
    • best seen on the mortise
  • The presence of hawkins sign is a reliable indicator that AVN is not presence
  • HOWEVER, the absence of hawkins sign does mean they have AVN
  • If you have concerns the best next step in an MRI
348
Q

What are options for surgical treatment of MDI

A
  • This should only be done after exhaustion of non-operative options
  • must address capsule +/- rotator interval
  • inferior capsular shift (capsule shifted superiorly)
    • plication of redundant capsule in a balanced fashion
  • rotator interval closure (open or arthroscopic)
    • produces the most significant decrease in range of motion in external rotation with the arm at the side
  • address any anterior or posterior labral pathology if present
  • thermal capsulorrhaphy (historical)
349
Q

Cervical kyphosis associated with larsen syndrome; what is the treatment?

A

posteior stabilization with halo for 3 months; prevents neurological deficit

early recognition is the key

350
Q

What are the zones of articular cartilage

A
  • Superficial zone (tangential zone)
    • Type II collagen orientation is parallel to joint
    • Has flattened chondrocytes, condensed collagen fibers, and sparse proteoglycans
    • only zone where articular cartilage progenitor cells have been found
  • Intermediate zone
    • Type II collagen has an oblique or random organization
    • Is the thickest layer with round chondrocytes, and abundant proteoglycan content
  • Deep layer (basal layer)
    • Type II collagen is perpendicular to joint and crosses tidemark; has the highest concentration of proteoglycans
    • Round chondrocytes arranged in columns
  • Tidemark
    • Is deep to the basal layer and separates the true articular cartilage from the deeper cartilage that is a remnant of the cartilage anlage, which participated in endochondral ossification during longitudinal growth in childhood.
    • The tidemark divides
      • the superficial uncalcified cartilage from the deeper calcified cartilage
      • division between nutritional sources for the chondrocytes
    • The tidemark is found only in joints
    • Most prominently in the adult and nongrowing joint
  • Subchondral Bone
351
Q

What are the transfers for a high median nerve plasy?

A
  • Sites of compression
    • ligament of struthers
    • lacertus fibrosus (under biceps)
    • head of pronator
    • FDS - subliminus bridge
    • accessory head of FPL (Gantzer)
  • Innervation
    • Median nerve
      • FDS, PL, Pronator teres, FCR
      • Palmar branch of the median cutaneous
        • thenar palm
      • AIN
        • FPL, FDP, Pronator
  • Restoration of Thumb Opposition (Opponensplasty)
    • EIP to APB (ie, Burkhalter)
    • FDS of ring finger to APB (ie, Royle- Thompson)
    • PL to APB (ie, Camitz)
  • Restoration of Thumb Interphalangeal Flexion
    • BR to FPL
    • ECRL to FPL
    • ECU to FPL
  • Restoration of Index Distal Interphalangeal Flexion
    • ECRL to FDP
    • side to side FDP
352
Q

What are injury patterns suggestive of PM of PL rotatory instability of the elbow?

A
353
Q

Signs associated with ulnar neuropathy

A

<!--StartFragment-->

  • sensory loss
  • hypothenary atrophy
  • 1st web space atrophy
  • ulnar claw hand
  • tinel’s sign
  • weakened grasp
    • from loss of MP joint flexion power
  • weak pinch
    • from loss of thumb adduction (as much as 70% of pinch strength is lost)
  • **Froment sign **
    • compensatory thumb IP flexion by FPL (AIN) during key pinch
    • compensates for the loss of MCP flexion by adductor pollicis (ulna n.)
    • adductor pollicis muscle normally acts as a MCP flexor, first metacarpal adductor, and IP extensor
  • **Jeanne sign **
    • compensatory thumb MCP hyperextension and thumb adduction by EPL (radial n.) with key pinch
    • compensates for loss of IP extension and thumb adduction by adductor pollicis (ulna n.)
  • Wartenberg sign
    • persistent small finger abduction and extension during attempted adduction secondary to weak 3rd palmar interosseous and small finger lumbrical
  • Masse sign
    • palmar arch flattening and loss of ulnar hand elevation secondary to weak opponens digiti quinti and decreased small finger MCP flexion

<!--EndFragment-->

354
Q

Peak blood supply during fracture healing?

A

2 weeks Normal by 3-5 months

355
Q

Complications associated with pediatric femur fracutre

A

<!--StartFragment-->

  • Leg-Length Discrepancy
    • most common complication
    • overgrowth
      • 0.7 - 2 cm is common in patients between the ages of 2 - 10 years at time of fracture
      • typically presents within 2 years of injury
    • shortening
      • is acceptable if less than 2 - 3 cm because of anticipated overgrowth
      • can be symptomatic if greater than 2 - 3 cm
      • temporary traction or internal fixation used to prevent persistent shortening
  • **Osteonecrosis (ON) of femoral head **
    • has been reported with piriformis and greater trochanter entry nails
    • femoral nailing through the piriformis fossa is contraindicated in adolescents with open physes because of the risk of osteonecrosis of femoral head
  • Nonunion
    • higher risk with load bearing devices
      • external fixator or submuscular plates
    • can occur after flexible intramedullary nailing in patients
      • aged over 11 years old
      • who weigh >49 kg (>108 lb)
  • Malunion
    • typical deformity is varus and flexion of the distal fragment
      • remodeling is greatest in sagittal plane (ie flexion/extension deformity)
      • remodeling does not occur with rotational malalignment and therefore must be corrected at the initial surgery
    • rarely symptomatic
  • Refracture
    • most commonly seen after external fixator removal
    • highest risk in transverse and short oblique fractures
    • less likelihood of secondary callus formation

<!--EndFragment-->

356
Q

important findings on ncs

A
  • Decrease in amplitude = axonal loss
    • occurs with more severe compression
    • will also be detected by EMG
  • Increase in latency = demyelination
    • typically occurs with a compression injury (cubital tunnel)
  • Sensory is affected first
  • Late waves
    • H - reflex is a measure of the achilles
      • absence is an indication of peripheral or S1 radiculopathy
    • F - waves
      • more indicitive of proximal pathology
      • thoracic outlet
  • Factors that can affect NCS
    • Temperature, age, height
357
Q

What is congential clasp thumb?

A
  • fixed flexion/adduction deformity
  • associated with contracture of the first web space and thenar muscls
  • associated with arthrogryposis and other anomalies
358
Q

What is the reduction maneuver for a bennet fracture?

A
  • Forces on fracture
    • Volar oblique ligament
    • Abductor pollicis longus (PIN) - base
    • Extensor pollicis longus (PIN) - base
    • Adductor pollicis (Ulnar) - shaft
  • Reduction maneuver
    • Traction
    • Abduction
    • Extension
    • Pronation
359
Q

What are the restraints to the shoulder

A
  • SGHL
    • restraint to inferior translation at 0° degrees of abduction (neutral rotation)
  • MGHL
    • resist anterior and posterior translation in the midrange of abduction (~45°) in ER
  • IGHL
    • posterior band IGHL
      • most important restraint to posterior subluxation at 90° flexion, abduction and IR
      • tightness leads to internal impingement and increased shear forces on superior labrum (linked to SLAP lesions)
    • anterior band IGHL
      • primary restraint to anterior/inferior translation 90° abduction and maximum ER (late cocking phase of throwing)
        • anchors into anterior labrum
        • forms weak link that predisposes to Bankart lesions
    • superior band IGHL
      • most important static stabilizer about the joint
      • 100% increased strain on superior band of IGHL in presence of a SLAP lesion
  • Coracohumeral ligament (CHL)
    • limits posterior translation with shoulder in flexion, adduction and internal rotation
    • limits inferior translation and external rotation at adducted position
360
Q

Camptodactyly - pathology and treatment options

A
  • Pathology - usually sporatic
    • abnormal lumbrical insertion/origin
    • abnormal (adherent, hypoplastic) FDS insertion
    • other less common causes include
      • abnormal central slip
      • abnormal extensor hood
      • abnormal volar plate
      • skin, subcutaneous tissue, or dermis contracture
  • Non-operative treatment is favoured
    • most will respond to passive splinting
  • FDS tenotomy +/- FDS transfer
    • reserved for cases of progressive deformity leading to functional impairment
    • technique
      • must address all abnormal anatomy
      • passive (correctable) deformities
      • FDS tenotomy, or
      • FDS transfer to radial lateral band if full active PIP extension can be achieved with MCP flexion
  • osteotomy vs. arthrodesis
    • severe fixed deformities
361
Q

What are indications for ankle arthroscopy

A

<!--StartFragment-->

Assymetric joint space narrowing is not an indication to do ankle arthroscopy

  • osteochondral lesions of the talus
  • microfracture of OCD
  • debridement of post-traumatic synovitis
  • ATFL anterolateral impingement
  • AITFL anterolateral impingement
  • resection of anterior tibiotalar spurs
    • such as anterior bony impingement
  • os trigonum excision
  • removal of loose bodies
  • cartilage debridement in conjunction with ankle fusions

<!--EndFragment-->

362
Q

List the modulus of elasticity of materials from highest to lowest

A

Ceramic (Al2O3)
Alloy (Co-Cr-Mo)
Stainless steel
Titanium
Cortical bone
Matrix polymers
PMMA
Polyethylene
Cancellous bone
Tendon / ligament
Cartilage

363
Q

Anatomy and causes of thoracic outlet syndrome

A
  • F>M
  • caused by compression of neurovascular bundle as it passes over the first rib or through the scalene muscle
  • causes include
    • scalene muscle abnormalities
    • scapular ptosis
    • clavicle and first rib malunion
    • cervical rib
    • vertebral transverse process
  • causes in athletes
    • fibromuscular bands
    • abnormal pectoralis minor
    • repetitive shoulder use
    • extreme arm positions
    • weightlifting, rowing, swimming
364
Q

Stabilizers of the DRUJ

A
  • Primary
    • volar radioulnar ligaments
      • volar translation
      • supination
    • dorsal radioulnar ligaments
      • dorsal translation
      • pronation
    • trigangular fibrocartilage
  • Secondary
    • Boney = 20%
      • ulnar head
      • sigmoid notch
    • interosseous membrane
      • constrains dorsal radius translation
  • Note - ECU subsheath and volar ulnocarpal ligaments do not play a role in stabilization
365
Q

What is the characteristic pattern of this injury?

A

lumbar burst fracture

  • centripital disruption of the vertebral body
  • anterior wedging
  • laminar fractures abut the spinous process
  • increased interpedicular distance
  • retropulsion of the superior endplate
366
Q

Pathology of this hand and tests

A
  • intrinsic plus
  • causes
    • trauma
    • rheumatoid arthritis (secondary to distal joint malalignment)
  • pathology
    • spastic intrinsics
      • lead to flexion of the MCP and extension of the IP joints
    • weak EDC
      • fails to provide balancing extension force of MCP
    • weak FDS & FDP
      • fails to provide balancing flexion force on PIP and DIP
  • ​​Bunnell test - for intrinsic
    • ​diff between intrinsic and extrinsic
    • test is positive when there is less PIP flexion with MCP extension than when the MCP is flexed
367
Q

What are the indications pin the other side?

A

endocrinopathy

<10 with open triradiate

lack of ability to follow-up

368
Q

What is true about reamed tibial nails?

A
  • SPRINT trial
    • improved stability with use of a larger nail
    • more biomechanically stable construct
    • Improved outcomes with reamed nails for closed fractures
    • No difference with open fractures
369
Q

What are the characteristics of thoracic outlet syndrome

A
  • clavicle, first rib, subclavius muscle, costoclavicular ligament, and anterior scalene muscle
    • junction of lower trunk and first rib
  • Brachial plexus and subclavian vessels travel between anterior and middle scalene
    • more often neuro compression occurs
    • rarely vascular compression occurs; this requires heparinizaiton and surgical treatment
  • Physical
    • Wright
      • abduction and external rotation with the neck rotated away leads to loss of pulse and reproduction of symptoms
    • Adson
      • extension of the arm with the neck extended and turned towards the affected side may result in loss of radial pulse or reproduction of symptoms with inhaling
    • Roos
      • hands repeatedly opened and closed while holding them overhead for 1 minute can reproduce symptoms
  • CXR
    • most common finding is a cervical rib
    • assess for pancoast tumor
370
Q

What is the most common brachial plexus injury?

A

Erbs (C5/6); waiter’s tip; best prognosis

371
Q

What is the heritance pattern of soft tissue sarcomas

A
  • usually result from translocations that occur as a result of lack of tumor suppression
  • spontaneous mutation of p53 is much more common than inheritable
    • Li-Fraumeni is an example of a heritable p53 disorder
372
Q

What are the soft tissues disrupted with PIP dislocation and what are the blocks to reduction.

A
  • Collaterals may also be disrupted
  • Dorsal
    • Volar plate - loose the static restraint
  • Volar
    • lateral bands/central slip

The third is refered to as a pilon fracture

373
Q

What is the pathology regarding CP thumb in palm deformity

A
  • CP - Thumb in palm/clasp thumb
    • many babies hold thumb in palm
    • most are not associated with a syndrome
  • contracture of first web and tight fascial bands in the first dorsal interosseous and adductor aponeuroses
  • Thumb-in-palm deformities are complex, and require careful assessment before the appropriate treatment can be recommended.
  • Principles of treatment:
    • release of contractures of the skin, joints, and spastic muscles
    • stabilization of the joints (MCP is usually unstable)
    • augmentation of the weakened muscles
374
Q

Hypovolemic shock, ATP depleted. What happens intracellular?

A

Increased sodium, decreased potassium, increased water

Cellular level – O2 demand > O2 supply – cellular hypoxia

375
Q

What is true about pediatric monteggia fractures?

A

plastic deformity of the ulna

peaks 4-10 yo

376
Q

What is condensing osteitis

A
  • sclerosis and englargement of the medial clavicle
  • women, 30-40, unilateral
  • most respond to NSAIDS
377
Q

What are the indications for an endocrine work-up for SCFE?

A

< 5% percentile growth

<10yo

378
Q

Associated conditions with this deformity

A
  • Note that these are not mutally exclusive and you should always screen for all of them; see baby below with thumb hypoplasia, fanconi’s anemia; also has several features of VACTERL
    • ​CBC
    • echo
    • Renal U/S
    • full head-toe exam
  • These patients should be treated with a team based approach - invovle genetics, pediatrics and all necessary specialties
  • TAR
    • autosomal recessive condition with thrombocytopenia and absent radius
    • thrombocytopenia fluctuants over time
      • exaccerbated by infection, stress
    • presents in first week of life
      • purpura, petechiae
      • intracranial hemmorhage - fatigues, lethary, difficulty feeding
    • low platlets, normal chormosomes
    • Basic first aid, platelet transfusion
    • different in that thumb is typically present
  • Fanconi’s anemia
    • autosomal recessive condition with aplastic anemia
    • can get hemmorhage, infection, neoplasia
    • Work-up
      • CBC - pancytopenia or complete aplasia
      • Diagnosis - Fanconi screen and chromosomal breakage test to screen
    • treatment is bone marrow transplant
  • Holt-Oram syndrome
    • autosomal dominant condition characterized by cardiac defects
      • ASD most common
    • diagnoses with blood test and genetics
  • VACTERL Syndrome
    • vertebral anomalies
    • anal atresia
    • cardiac abnormalities
    • tracheoesophageal fistula
    • renal agenesis
    • limb defects
  • VATER Syndrome
    • vertebral anomalies, anal atresia, tracheoesophageal fistula, esophageal atresia, renal agenesis)
379
Q

What is Li-Fraumenni syndrome

A
  • AD inheritance
  • linked to P53 inactivity
  • linked to rhabdomyosarcoma in kinds
  • also known as Sarcoma, breast, leukaemia and adrenal gland (SBLA) syndrome.
380
Q

What work-up do you need to do when you see thumb hypoplasia?

A

Blood work (anemia, thrombocytopenia)

Echo/asceulation

Renal U/S

Abdomenal U/S

381
Q

What are the important clinic aspects of multiple enchondromas (not MHE)

A
  • Ollier’s disease (multipe enchondromatosis)
    • no genetic predisposition has been identified
    • multiple enchondromas in long bone diaphysis
    • 10-30% chance of malignant transformation into chondrosarcoma
    • however associated with high rate of both skeletal and non-skeletal malignancies
    • involved bones are dysplastic
  • Maffucci’s syndrome
    • no genetic predisposition has been identified
    • multiple enchondromas and soft-tissue angiomas
    • 100% chance of malignant transformation into chondrosarcoma
    • increased risk of visceral malignancies (astrocytoma, GI malignancy)
382
Q

What is the appropriate tibial tunnel placement for ACLR?

A

proper placement

center of tunnel entrance into joint should be 10-11mm in front of anterior border of PCL insertion

tunnel trajectory of < 75° from horizontal

obtain by moving tibial starting point halfway between tibial tubercle and posterior medial edge of tibia.

anterior misplacement

leads to knee that is tight in flexion with impingement in extension

posterior misplacement

leads to ACL that will impinge on posterior condyle

383
Q

What are the associated mobility for myelomeningocele levels?

A
384
Q

What are factors that lead to keinbochs disease?

A

male, 20-40

repetative injury

I pattern vascularity

ulnar negative variance

385
Q

What is ture about congenital hallux varus?

A

Atavastic great toe

short, thick, first MT

associated with polydactyly

Often need to syndactylize to second toe to control deformity

MT osteotomy +/- lengthening

release addutor hallucis

386
Q

What is not important when evaluating an outcome measurement?

A

cost effectiveness

(content, methodology and clinical utility are all important)

387
Q

What is the definative treatment for a parosteal osteosarcoma?

A
  • wide resection
  • no role for chemo unless there is a high grade component
  • 30-40 yo
  • surface of bones, usually the distal femur
  • less aggressive than periosteal osteosarcoma
388
Q

What is the best orthotic for an ankle fusion?

A

Heel to toe rocker

389
Q

What is typical and how do you treat fibrosarcoma?

A

> 50 yo (worse prog over 40)

pain, swelling, path #

distal femur, proximal tibia

XR - purely lytic wth moth eatten appearance, no evidence of codman’s triangle or osseous matrix

Treat as if osteosarc

Chest XR - rule out lung mets

Local staging with MRI

Chemo - doxyrubicin and cysplatinin

Re-stage

Limb salvage surgery

Chemo

390
Q

What is the criteria for causation

A
  • ACCESS PTB
  • Analogy: The effect of similar factors may be considered.
  • Consistency: Consistent findings observed by different persons in different places with different samples strengthens the likelihood of an effect.
  • Coherence: Coherence between epidemiological and laboratory findings increases the likelihood of an effect. However, Hill noted that “… lack of such [laboratory] evidence cannot nullify the epidemiological effect on associations”
  • Experiment:“Occasionally it is possible to appeal to experimental evidence”
  • Strength: A small association does not mean that there is not a causal effect, though the larger the association, the more likely that it is causal
  • Specificity: Causation is likely if a very specific population at a specific site and disease with no other likely explanation. The more specific an association between a factor and an effect is, the bigger the probability of a causal relationship
  • Plausibility: A plausible mechanism between cause and effect is helpful (but Hill noted that knowledge of the mechanism is limited by current knowledge)
  • Temporality: The effect has to occur after the cause (and if there is an expected delay between the cause and expected effect, then the effect must occur after that delay).
  • Biological gradient: Greater exposure should generally lead to greater incidence of the effect. However, in some cases, the mere presence of the factor can trigger the effect. In other cases, an inverse proportion is observed: greater exposure leads to lower incidence
391
Q

What are the blocks to medial subtalar dislocation?

A

<!--StartFragment-->

peroneal tendons
extensor digitorum brevis
talonavicular joint capsule

<!--EndFragment-->

392
Q

What are the forms of myelomeningocele

A

<!--StartFragment-->

spinal bifida oculta

defect in vertebral arch with confined cord and meninges

meningocele

protruding sac without neural elements

myelomeningocele

protruding sac with neural elements

rachischisis

neural elements exposed with no covering

<!--EndFragment-->

393
Q

What is the approach to continued hypotension in the setting of a pelvic fracture?

A

Angio for embolization

394
Q

What is sternoclavicular hyperostosis and what is associated with it?

A

SAPHO

pustulitic arthro-arthritis

soft tissue ossification between the clavicles

palmoplantar pustolosis and severe acne

Males 40-50, severe pain and redness over SC joint

often bilateral

395
Q

What is the most common location for compression of the suprascapular nerve?

A
  • suprascapular notch apparently more common
    • due to confinement and limited mobility
    • nerves to supraspinatous are adhered to periosteium
    • leads to more functional deficiet
  • Other site is spinoglenoid notch
    • IS is more compenstated by deltoid
  • Etiology
    • cysts
    • transverse scapular ligament
    • fracture callus
    • Can be associated with
      • RC tear, AC OA, adhesive capsulitis, GH OA, parsonage turner syndrome
  • XR - stryker notch view
  • EMG/NCS will confirm pathology of location of the entrapment
  • Get MRI and treat pathology
    • trace the nerve
    • look for mass occupying lesions
    • compare SS and IS atrophy
396
Q

What is the order of closure for an ankle physis?

A

central, anteromedial, then circles laterally

397
Q

What are the deformities associated with CVT?

A

oppositve to clubfoot except achilles

flat foot

abducted

valgus

equinus

talus is vertical and not articulating with navicular; need to get platarflexion views to R/O the more common calcaneovalgus

Achilles, peroneals are tight, tib ant and post are not functioning

398
Q

Diagnosis? Classification? Indications/Options for surgery?

A

Radial Head Fracture

  • Physical
    • check ROM
      • 30-130 flex-ex
      • 45-45 sup-pro
    • check the pin
  • Imaging
    • ​can get a greenspan view to help assess
      • 45 deg oblique center over the RC joint
    • CT
  • Classification - Mason Allen
    • Type 1 - nondisplaced
    • Type II - partial head fx
    • Type III - complete head
    • Type IV - radial head fx with elbow dislocatoin
  • short period of immobilization followed by early ROM
    • Nondisplaced, stable (30-130)
    • no mechanical block
    • complications
      • nonunion
        • new studies have shown a higher rate of nonunion in nonoperative managmeent than previously expected
  • ORIF
    • Indications
      • Simple fracture
        • Mechanical block
    • technique
      • plate in safe zone
      • countersink screws in articular surface
  • Fragment Excision (partial excision)
    • indicated if fragment
    • Cannot be a part of the PRUJ
    • complications
      • even small fragment excision may lead to instability
  • radial head replacement
    • indications
      • for comminuted fracture with 3 or more fragments
    • complications
      • overstuffing of joint that leads to capitellar wear problems and malalignment instability
  • radial head resection
    • indications
      • older patients with limited demands
      • in a delayed setting for continued pain of an isolated radial head fracture
    • contraindications
      • forearm ligament injury (identify with radius pull test)
      • coronoid fracture
      • MCL deficiency
    • complications
      • muscle weakness
      • wrist pain
      • valgus elbow instability
      • heterotopic ossification
      • arthritis
399
Q

What is the dalbet classification of pediatric femoral neck fractures?

A
  • Type I Transphyseal (10%)
    • IA - without dislocation of epiphysis from acetabulum
    • IB, with dislocation of epiphysis
    • Risk of ANV = 90-100%
  • Type II Transcervical (40-50%)
    • AVN - 50%
    • Nonunion - 15%
  • Type III Basicervical (30-35%)
    • AVN - 25%
    • Nonunion - 15-20%
  • Type IV Intertrochanteric (10-20%)
    • AVN - 10%
    • 5%
400
Q

Treatment of MCL tears

A
  • Some will recommend in combination ACL - rehab to get ROM, then reconstruct ACL and if still valgus instability reconstruct the MCL - highly controversial as to what you do
  • Nonoperative
    • treat in hinged knee brace for 6-8 weeks
    • only knee ligament injury the body can heal independently
    • proximal ruptures have better healing potential than distal ruptures
    • rehabilitation
      • early ROM and functional rehab (immediate quads) in athletes
      • return to play depends on discomfort and residual instability
        • Grade I: 2 weeks
        • Grade II: 3 weeks
        • Grade III: full extension for 2 weeks followed by 6 to 8 weeks of total rehab (must ensure it is isolated MCL)
    • medications
      • ibuprofen shown to not be deleterious to MCL healing
      • new studies have shown that platelet-derived growth factor (PDGF) improves healing
  • Operative
    • indicated only in Grade III injuries as part of multiple ligament knee injuries (with ACL and/or PCL/PLC)
      • tibial avulsion in athlete
      • grade III with valgus alignment
      • associated fractures (plateau or avulsion)
      • intra-articular intrapment
    • treatment options include
      • treat MCL nonoperatively, wait for it to heal, then reconstruct ACL
      • primary repair of all torn structures
        • use sutures, suture anchors, or screws with washers for avulsions of the MCL origin or insertion
      • MCL reconstruction
        • can use allograft or semitendinosus autograft
        • attach to anatomic origin and insertion
      • tension in 30° knee of flexion after PCL, ACL, and lateral side have been tensioned
    • timing of repair
      • address multiple ligament knee injuries as soon as safely possible
      • allow 2-3 weeks for capsule to heal to allow arthroscopy and still permit repair of posterolateral structures
    • postoperative
      • block full knee extension to protect repair for 3 weeks, then full ROM and WBAT at 3 weeks
      • progressive activity starting at 6 weeks
  • complications
    • stiffness
      • must counsel patient increased risk of loss of motion
401
Q

What is the salvage options for an axillary nerve palsy?

A

GH arthrodesis

402
Q

What approach is best used for a coronal shear of the humerus?

A

Lateral approach - kocher distally to see LCL, mobile wad and extensors proximally; the large the peice the more anterior you need to get (can use a homen under brachialis and capsule to get across the humerus)

On the rare occasion that you can’t see across you can do a concurrent flexor/pronator split medially, or use an olecranon osteotomy (although you don’t see much joint anteriorly this way)

403
Q

What loss in the forearm will develop with a 10 deg radius malunion

A

Rockwood and Green Ch 27

They observed a significantly greater loss of ROM in forearms with middle-third deformities than with distal-third deformities, with more supination being lost than pronation. They also observed a significant decrease of ROM with 15° of angulation

404
Q

vertebral body forms from?

A

mesoderm

405
Q

In a stress-strain diagram, what does the area under the curve represent?

A

toughness - engery absorbed until failure

406
Q

What is the essential pathoantomy for recurrent shoulder instability

A

Bankhart lesion

407
Q

What are the components of proteoglycans?

A
  • makes up 10 to 15% of cartilage
  • functions to provide compressive strength and attract water
  • produced by chondrocytes
  • composed of GAG subunits
    • chondroitin sulfate
    • keratin sulfate
408
Q

Compare Humeral shaft IM nail and plates

A
  • Nerve injury same risk
  • Reports of higher rates of nonunion with IM nail
  • reports of better function with plate
409
Q

Closure of what physis correlates with peak growth velocity?

A

olecranon

410
Q

What are the common tumours of the posterior elements of the spine?

A

ABC

osteoblastoma

osteoid osteoma

osteochondroma

411
Q

What are the advantages of treatment GCT with cement

A
  1. immediate mechanical stability
  2. low cost
  3. ease of use
  4. no donor site morbidity
  5. no disease transmission
  6. easier detection of recurrence
  7. exothermic reaction leaves a rim of necrotic bone
412
Q

Boundaries and zones of guyons canal?

A
  • Boundaries of Guyon’s canal
    • Floor
      • Transverse carpal ligament, hypothenar muscles
    • Roof
      • Volar carpal ligament
    • Ulnar border
      • Pisiform and pisohamate ligament, abductor digiti minimi muscle belly
    • Radial border
      • Hook of hamate
  • Zones of Guyon’s canal
    • Zone 1
      • Proximal to bifurcation of the nerve
      • Ganglia and hook of hamate fractures
      • Mixed motor and sensory
    • Zone 2
      • Surrounds deep motor branch
      • Ganglia and hook of hamate fractures
      • Motor only
    • Zone 3
      • Surrounds superficial sensory branch
      • Ulnar artery thrombosis or aneurysm
      • Sensory only
413
Q

Strongest pullout strength in tendon repair

A

pulvertaf

414
Q

What is the classification for a coronoal shear fracture of the humerus?

A

I - capitellum

II - avulsion capitellum

III - comminuted capitellum

IV - trochlea (double arc sign)

415
Q

Diagnosis? Differential? Treatment?

A

Torticollis

  • Deformity in the SCM
    • head tilts towards the affected side and rotates away
  • Pathophysiology
    • some studies suggest association with intrauterine compartment syndrome of SCM
    • non-SCM causes include
      • ophthalmologic
      • vestibular
      • congenital
      • traumatic
      • tumors
  • Associated conditions
    • Packing disorders
      • DDH (5 - 20% association)
      • metatarsus adductus
    • traumatic delivery
    • plagiocephaly (head assymetry)
    • congenital atlanto-occipital abnormalities
  • Differential
    • Rotatory atlanto-axial instability / Grisel’s disease
    • Klippel-Feil syndrome
  • Imaging
    • ​XR/CT
      • if no palpable mass
      • to rule out AARD/klippel-fiel
    • US can show lesion in SCM
  • ​Treatment
    • passive stretching
      • condition present for less than 1 year
      • limitation less than 30°
      • should include lateral head tilt away from the affected side and chin rotation toward the affected side (opposite of the deformity)
      • 90% response with-in one year
    • Z plasty lengthening or distal bipolar release of SCM
      • > 1 year
      • > 30°
416
Q

Recurrent laryngeal is a branch of what nerve

A
  • vagus
  • loops around the aorta on the left and subclavian on the right
  • historically less risk on the left but new studies to show no difference
  • Treatment of injury
    • monitor
    • if no return in 6 weeks refer to ENT for assessment and teflon injection
417
Q

What is the most common sarcoma of the hand

A

chondrosarcoma

418
Q

What is true about ACL association with OA?

A

associated with increased OA

ACL recon does not decrease OA risk

chronic ACL tear = medial meniscus

419
Q

What does it mean if you have a correctable pes planus foot with toe walking?

A

normal subtalar motion

420
Q

Medications that cause osteoporosis

A

steroids
thyroid meds
antacids
anti-convulsants (Vit D metabolism)
PPI’s
Tetracycline
Loop diuretics
Lithium
Methotrexate

**Diclofenac does not

421
Q

What is the incidence of metaplasia and malignant transformation of a chronic draining sinus?

A

0.2-1.6%

Squamous cell carcinoma = Marjolin’s ulcer

422
Q

What are side effects of bisphosphanates in children?

A

acute fever

transient hypocalcemia, hypophosphatemia, hypomagnesemia

Oversuppression of bone remodelling

NO growth delay

NO fracture healing issues

Some difficulty healing osteotomy

423
Q

What are the nerve transfers for a low median nerve palsy?

A

FDS to APB (bunnell opponensplasty)

EIP to APB

424
Q

When should you not primarily nail a tibia

A
  • Open segmental fracture, irrespective of the size of the wound
  • Gunshot wounds -high velocity and short-range shotgun injuries
  • Open fracture with neurovascular injury
  • Farm injuries, with soil contamination, irrespective of the size of the wound
  • Traumatic amputations
  • Open fractures over 8 hours old
  • Mass casualties; eg, war and tornado victims
425
Q

What is the most common nerve injury in elbow arthroscopy and what portal is most at risk?

A

ulnar (then radial)

proximal medial

426
Q

What are variables associated with failure of non-op DR#?

A

>80 yo

>2mm ulnar variance

metaphyseal comminution

dependant patient

NOT dorsal angulation

427
Q

What diseases have occular associations?

A

Homocysteinuria has inferior lens dislocation
Marfan’s has superior lens dislocation
Neurofibromatosis has Lisch nodules in the iris.

428
Q

Iatrogenic causes of this deformity

A
  • overcorrection of 1st IMA
  • excessive lateral capsular release with overtightening of medial capsule
  • overresection of medial first metatarsal head
  • lateral sesamoidectomy
429
Q

What is the best managment of a femoral neck fracture

A

3 cannulated screws

inverted triangle

430
Q

What is the appropriate wrist position for an unstable DRUJ for galeazzi fracture?

A

Overall - supnation is the most stable position for the DRUJ

dorsal - supnation

volar - pronation

LAC - 6 weeks

431
Q

What are the 4 types of RA triggering

A
  • Type 1: similar to nonrheumatoid stenosing tenosynovitis, in which the tendons catch at the first annular pulley during flexion secondary to small, localized hyperproliferation of the synovium.
  • Type 2: the nodules form in the distal palm and cause the finger to lock in flexion. In
  • type 3: nodules on the flexor digitorum profundus (FDP) tendon near the second annular pulley (over the proximal phalanx) lock the finger in extension.
  • Type 4: trigger finger results from generalized tenosynovitis within the fibroosseous canal. Active motion is more restricted than passive motion, and contracture and stiffness result.
432
Q

What are the differences in outcomes for a garden 3/4 for femoral neck fractures comparing internal fixation and arthroplasty?

A
  • pain scores better in arthroplasty
  • revision rate lower in fixation
  • mortality the same
  • blood loss lower for internal fixation
433
Q

Diagnosis? Blocks to reduction? Treatment?

A
  • Volar IP dislocation
    • always has a central slip rupture
  • late deformity
    • Boutinerre - from central slip rupture
  • Blocks to reduction
    • ​one collateral
    • central slip and lateral band
      • _​_button holes between the two
  • ​Treatment
    • ​buddy tape for 6-8 weeks if reducible
      • if fracture < 40%
      • no associated fracture
    • ORIF or CRPP
      • >40% articular surface
434
Q

What is the anatomy of the medial circumflex artery

A

Comes underneath quads femoris, travels superiorly in front of the ERs, except obturator externus because it is a more antereiror structure

435
Q

What spondylolisthesis are more likley to progress?

A

dysplasic (and more neuro)

>50% slip

>50 deg slip angle

436
Q

How does ulnar variance change with movement?

A
  • Normally takes 20% loads
  • positive - pronation
  • negative - supnation
437
Q

What do you need to consider in the treatment of syndactyly of the hand

A
  • Timing
    • syndactyly
      • perform at ~ 1 year of age
      • controversial
    • acrosyndactyly
      • perform in neonatal period
    • if multiple digits are involved perform procedure in two stages to avoid compromising vasculature
      • early surgery allows this
      • thumb and index at
    • release digits with significant length differences first to avoid growth disturbances
  • technique
    • zigzag flaps are created during release to avoid longitudinal scarring
    • a nailbed can be re-created in complete syndactyly
  • complications
    • Web creep
      • most common complication of surgical treatment
      • reconstruct web space with local skin flaps
    • Nail deformities
    • Scar
    • Avascular digit
      • _​_never operate threw both sides of the digit
438
Q

How does treatment of the fibula affect the outcome of distal tibia fractures

A
  • With IM nails
    • improved stiffness
    • improved alignment
    • increased risk of nonunion
      • avoid if causes tibial fracture gapping
  • Absolute indications
    • evidence of syndesmotic injury
    • medial tibial plating
      • to avoid valgus
439
Q

What are the 5 forms of lubrication

A
  • elastohydrodynamic
    • main mechanism during dynamic joint function
    • elastic deformation of articular surfaces
    • thin films of lubricant separate the surfaces
  • boundary (slippery surfaces)
    • bearing surface is non-deformable
    • lubricant only partially separates surfaces
    • superficial zone proteins have a role in this lubrication mechanism
  • boosted (fluid entrapment)
    • concentration of lubricating fluid in pools
    • trapped by regions of bearing surfaces that are making contact
  • hydrodynamic
    • fluid separates surfaces when one surface is sliding on the other
  • weeping
    • fluid shifts out of articular cartilage in response to load
    • surfaces separated by hydrostatic pressure
440
Q

What is the best position for bilateral elbow fusion?

A

65 and 110

441
Q

What tendon is used to augment the achilles for >50% debridement for achilles tendonitis

A

FHL

442
Q

Factors associated with DHS cutout

A

NOT Sex

NOT medical status

Tip-apex > 25mm

increased age (controversial)

unstable fracture

poor reduction

Higher (150) deg plate

443
Q

What are the characteristic of a SLAP lesion

A
  • helps to maintain the stability of the joint
    • strains the IGHL without it
  • Buford complex is a normal anatomical variant
  • Pathoanatomy
    • forceful traction
    • repetative throwing
    • direct compression
  • Physical exam
    • no specific exam is useful
    • O’briens, yeargson and speed can be helpful
    • assess the cuff
    • Assess for associated spinoglenoid compression
444
Q

What is the pathology with anticonvulsant associated osteomalacia?

A
  • Inability to hydroxylate 25-VitD
  • Accentuated by
    • inactivity
    • poor intake vit D
    • reduced exposure to sunlight
445
Q

What adjunts can be used with loosening a cast to help prevent compartment syndrome

A

Correct hypotension

correct anemia with transfusion

DON’T give O2

446
Q

What is the treatment of a chordoma?

A
  • most common presentation is pain
  • en bloc resection with tumor free margins
  • no role for chemo or RT because is a slow growing tumour
  • rads can be used if there is any concern at all about recurrence or positive margins
447
Q

What is true about acute nailing of open tibia fractures

A
  • No difference in reamed nails with regard to infection, nonunion or need for additional procedures
    • not suggested in grossly contaminated wound, delay to treatment, gustilo IIIb
  • Gapping is the number one reason for nonunion
  • BMP-2 has been shown to have decreased time to union, decreased need for secondary procedures in all fractures and decrease infection in Gustilo III
  • With regards to plating an open tibia - the most common reason they progress to nonunion is periosteal stripping
448
Q

Pediatric proximal humerus fracture; reduction maneuver and likley block to reduction?

A

90 deg abduction, 90 deg ER

LHB most common

capsule, periosteium

449
Q

Diagnosis? Important Concepts?

A
  • Epiphyseal, benign aggressive
    • can get cortial thinning with neocortex and soft tissue invasion​
    • destruction caused by activation of osteoclasts
  • 30-40 yo (ABC is 20)
  • Location
    • ​50% knee
    • 10% sacrum
    • distal radius
  • Medical treatment
    • ​bisphosphanates
    • denosemab
  • extensive curettage and reconstruction (with adjuvant treatment)
    • challenge - remove lesion while preserving joint and providing support to subchondral joint
    • extensive exterioration (removal of a large cortical window over the lesion) is required
    • 10-30% recurrence with curettage alone verses 3% with adjuvant treatment (phenol, hydrogen peroxide, argon beam, etc)
    • can fill lesion with bone cement or a_utograft/allograft bone_
  • hand lesion treatment is controversial
    • if no cortical breakthrough treat with curettage and cementing
    • if significant cortical breakthrough consider intercalary resection (with free fibular graft) vs. amputation
450
Q

What are the characteristics of carpal synostosis/coalition?

A
  • Blacks > whites
  • usually carpometacarpal fusion failure of cavitation 4-8 wks gestation
  • usually bilateral
  • 90% between lunate-triquetrum
451
Q

What is the heritance pattern of neurofibromatosis?

A

<!--StartFragment-->

  • 50% spontaneous
  • autosomal dominant (AD)
    • mutation in NF1 gene on chromosome 17q21
    • codes for neurofibromin protein
    • negatively regulates Ras signaling pathway
    • neurofibromin deficiency leads to increased Ras activity
    • affects Ras-dependent MAPK activity which is essential for osteoclast function and survival
  • neurofibromatosis is the most common genetic disorder caused by a new mutation of a single gene

<!--EndFragment-->

452
Q

How does the size of the intra-foramenal space change in the c-spine with movement?

A
  • increases with flexion and contralateral rotation
  • decreases with extension and ipsilateral rotation (ie; spurling test)
453
Q

Treatment indications for scoliosis associated with Duschnnes muscular dystrophy

A

<!--StartFragment-->

  • indications
    • curve > 20° in nonambulatory patient
      • (treat early, < 30° curve, before pulmonary function declines)
      • can wait slightly longer (Cobb ~ 40°) if patient is responding well to corticosteroids
    • FVC drops below 35%
    • rapidly progressive curve
  • techniques
    • extension to pelvis is controversial
  • Complications
    • malignant hyperthermia is common intraoperatively
      • pretreat with dantrolene
    • intraoperative cardiac events

<!--EndFragment-->

454
Q

Goals and indication for reconstruction of syndactyly

A
  • Goal is to have as many independant and functional digits as possible
    • deep and wide web
    • cover deficits with soft tissue
  • Indications
    • skin bridge that hinders use
  • Contraindications
    • super digit
    • complex digit that functions in unison
    • no active muscle control with functional digits
455
Q

Complications associated with tourniqute use

A
  • Local
    • post-op swelling and stiffness
    • delay in recovery of muscle power
    • compression neuropraxia
    • wound hematoma
    • wound infection
    • direct vascular injury
    • bone and soft tissue necrosis
    • compartment syndrome
  • Systemic
    • increased CVP
    • increased arterial HTN
    • Cariorespiratory compensation
    • cerebral infarct
    • altered acid-base balance
    • rhabdomyolysis
456
Q

Complications associated with SCH#

A

<!--StartFragment-->

  • Compartment syndrome
    • TIII, admit and monitor pre-op and post-op with Q2H compartment checks
    • Increased risk with worse deformity and vascular injury (even after repair)
    • Treatment - forearm fasciotomy
  • Pin migration
    • most common complication (~2%)
  • Infection
    • occurs in 1-2.4%
    • typically superficial and treated with oral antibiotics
  • Cubitus varus (gunstock deformity)
    • caused by fracture malunion
    • can lead to tarda ulnar nerve palsy
    • usually a cosmetic issue with little functional limitations - this is changing
    • Can lead to attenuation of the LUCL and eventually PLRI
      • Changes the force of the triceps (assymetric)
      • Osteotomy has good results
    • Corrective osteotomy is recommended if substantial deformity is present
      • Explain to parent risk of PLRI
      • Opening lateral wedge
    • Timing
      • >1 year after no improvement in ROM
      • When old enough to comply with PT
  • Recurvatum
    • common with non-operative treatement of Type II and Type III fractures
  • Nerve palsy
    • usually resolve with no need for treatment or investigation
  • Vascular Injury
    • Volkmann ischemic contracture
    • rare, but dreaded complication associated with supracondylar humerus fractures
    • more often as a result of brachial artery compression with treatment utilizing elbow hyperflexion and casting than true arterial injury
    • increase in forearm compartment pressures and loss of radial pulse with elbow flexed greater than 90°
    • rarely seen with CRPP and postoperative immobilization in less than 90°
  • Stiffness
    • Rare after casting or pinning
    • Will resolve by 6 months
    • Literature does not support use of PT

<!--EndFragment-->

457
Q

What is true about the blood supply to the humeral head

A
  • LHB can help locate the asecnding branch and localize the tuberosity
  • ascending branch is just posterior and lateral to the LHB
  • ascending branch of the anterior circumflex is the primary blood supply to the femoral head
  • the second supply is the posterior circumflex which wraps around the medial calcar
458
Q

Pathology of this XR?

A

VISI - volar intercalated segmental instability

  • LT ligament injury occurs with
    • wrist hyperextension or
    • extension and radial deviation
  • 2 important ligaments tears
    • ​lunotriquetral (intrinsic)
      • volar - strongest
      • dorsal - more important rotation
    • Dorsal radiocarpal (extrinsic)
    • volar long and short radioluntae ligaments - extrinsic that might also tear
  • scaphoid induces the lunate into further flexion while triquetrum extends

459
Q

Delayed healing of the UCL of the base of the 1st MCP is due to what fascia?

A

Adductor fascia (Stener lesion)

460
Q

Types of CRPS

A
  • Type I
    • noxious stimiulant
    • crush, soft tissue, immobilization in cast
  • Type II
    • defined nerve injury
  • Presentation
    • hypermia, edema, skin changes
    • temperature changes, sweating
  • Requires multi-modal and team approach with treatment started as soon as possible
  • Treatment
    • Continue aggressive PT
    • Medication
      • vitamin C
      • gabapentin
      • antidepressants, muscle relaxants
    • Nerve block
      • local injection into the nerve
    • Stellate ganglion blocks or spinal cord stimulator
      • should be reserved for refractory cases
461
Q

Diagnosis? Classification

A

Keinboch’s - Litchman Classification

  • Stage I
    • No visible changes on xray, changes seen on MRI
      • Immobilization
      • NSAIDS
  • Stage II
    • Sclerosis of lunate
      • Ulnar negative - ulnar shortening
      • Ulnar positive - Radial wedge, STT fusion
      • Distal radius core decompression
      • Revascularization procedures
  • Stage IIIA
    • Lunate collapse, no scaphoid rotation
      • Same as Stage II above
  • Stage IIIB
    • Lunate collapse, fixed scaphoid rotation
      • Proximal row carpectomy
      • STT fusion
  • Stage IV
    • Degenerated adjacent intercarpal joints
      • Wrist fusion
      • proximal row carpectomy
      • limited intercarpal fusion
462
Q

Classification of syndactyly

A
  • Simple
    • Only soft tissue involvement, no bony connections
  • Complex
    • Side to side fusion of adjacent phalanges
  • Complicated
    • Accessory phalanges or abnormal bones involved in fusion
  • Complete vs. Incomplete
    • Complete: syndactyly the skin extends to finger tips
    • incomplete: skin does not extend to fingertips
463
Q

What is the number one factor that will lead to stiffness following a phalanx fracture/dislocation?

A

Congruent joint surface OR only necessary if can’t do this with closed means

464
Q

Correlate growth velocity with menarche, apoyphyseal closure and the risser grade

A
465
Q

Treatment of a pediatric galeazzi fracture

A
  • Closed reduction in LAC for 6 weeks
  • ORIF with immoblization
    • loss of reduction at follow-up
    • unable to get a reduction at time of injury
  • More stable due to
    • thick periosteum
    • superior fracture remodeling capacity
    • the increased ligamentous strength and elasticity of the DRUJ
466
Q

What is the treatment for a pathological fracture of an enchondroma?

A

Wait for fracture to heal

467
Q

Options for treatment of hallux valgus

A
  • Nonoperative
    • shoe modification/ pads/ orthoses
    • silastic spacer
      • orthoses more helpful in patients with pes planus or metatarsalgia
  • Soft Tissue Procedure - modified McBride
    • never appropriate in isolation
    • in conjunction with medial eminence resection osteotomy
    • release medial capsule of 2nd MTP; can be sutured to lateral capsule of 1st
    • adductor hallucis is released and interposed
      • Can be left to scar down
    • Transverse intertarsal ligament is releases which attaches to the fibular sesamoid
    • Don’t resect the fibular sesamoid as was done with the original McBride or you will get hallux varus
  • distal metatarsal osteotomy
    • mild disease (HVA 20-40, IMA 10-13)
    • distal metatarsal osteotomies include
      • Chevron
      • biplanar Chevron
      • Mitchell
    • may be combined with proximal phalanx osteotomy
  • proximal metatarsal osteotomy
    • moderate disease (HVA >40°, IMA >13°)
    • proximal metatarsal osteotomies include
      • crescentric osteotomy
      • Broomstick osteotomy
      • Ludloff
      • Scarf
  • double (proximal and distal) osteotomy
    • severe disease (HVA 41-50°, IMA 16-20°)
    • DMAA > 15
    • Scarf + chevron
    • Lapidus + Akin
  • first cuneiform osteotomy
    • severe deformity in young patient with open physis
  • Lapidus procedure (1st metatarsocuneiform arthrodesis)
    • severe deformity
    • Metatarsus primus varus
    • hypermobile 1st tarsometatarsal joint
  • MTP Arthrodesis
    • Gout
    • Rheumatoid arthritis
    • Down’s syndrome
    • cerebral palsy
    • Severe DJD
    • Ehler-Danlos
    • Resection arthroplasty
  • proximal phalanx (Keller) resection arthroplasty
    • largely abandoned
    • still indicated in some elderly patient with reduced function demands
468
Q

What is the most common failure for a symes amputation?

A

posterior heel pad migration

others - skin sloughing, post tib comprimise

used to be an issue fitting a prosthesis due to limited space requiring lift of the contralateral limb - not an issue any more